Courses
Courses for Kids
Free study material
Offline Centres
More
Store Icon
Store

NCERT Solutions for Class 12 Maths Chapter 3 - Matrices

ffImage
Last updated date: 16th Apr 2024
Total views: 514.8k
Views today: 6.14k
MVSAT 2024

NCERT Class 12 Matrices Chapter 3: Complete Resource for Matrices

NCERT Solutions for Class 12 Maths Chapter-3 PDF is made available for download. The student can find the official PDF on the Vedantu’s website and download it from there. The Matrices Class 12 PDF relating to the complete solutions have been provided for Chapter 3 of Class 12 NCERT Maths. You can find everything related to Chapter 3 Maths Class 12 there. The Matrices Class 12 NCERT Solutions PDF download has been developed by esteemed academicians who are in the field for quite some time. In addition to this, if anyone faces any doubts concerning CH 3 Maths Class 12, they can go to the website and drop in their queries and download the Matrices Class 12 NCERT PDF version. The helping team will get back to the student in case of doubts and clear that off in a very efficient manner.


Class:

NCERT Solutions for Class 12

Subject:

Class 12 Maths

Chapter Name:

Chapter 3 - Matrices

Content-Type:

Text, Videos, Images and PDF Format

Academic Year:

2024-25

Medium:

English and Hindi

Available Materials:

  • Chapter Wise

  • Exercise Wise

Other Materials

  • Important Questions

  • Revision Notes



Matrices Chapter at a Glance - Class 12 NCERT Solutions

  • A matrix is an ordered rectangular array of numbers or functions.

  • A matrix having $m$ rows and $n$ columns is called a matrix of order $m \times n$.

  • $\left[a_y\right]_{m \times 1}$ is a column matrix.

  • $\left[a_h\right]_{\mathrm{b} x}$ is a row matrix.

  • An $m \times n$ matrix is a square matrix if $m=n$.

  • $A=\left[a_{i j}\right]_{\text {nex }}$ is a diagonal matrix if $a_{i j}=0$, when $i \neq j$.

  • $A_y=\left[a_{i j}\right]_{n \times n}$ is a scalar matrix if $a_{i j}=0$, when $i \neq j, a_{i j}=k,(k$ is some constant $)$, when $i=j$.

  • $A=\left[a_y\right]_{m \times n}$ is an identity matrix, if $a_{i j}=1$. when $i=j, a_{i j}=0$, when $i \neq j$.

  • A zero matrix has all its elements as zero.

  • $A=\left[a_i\right]-\left[b_y\right]-B$ if

(i) A and B are of asme order,

(ii) $a_{i j}-b_{i j}$ for all possible values of $i$ and $j$.

  • $k a=k\left[a_y\right]_{m \times n}-\left[k\left(a_y\right)\right]_{m \times n}$

  • $-A=(-1) A$

  • $A-B=A+(-1) B$

  • $A+B=B+A$

  • $(A+B)+C=A+(B+C)$, where $A, B$ and $C$ are of aame order.

  • $k(A+B)=k A+k B$, where $\mathrm{A}$ and $\mathrm{B}$ of asme order, $k$ is constant.

  • $(k+l) A=k A+l A$, where $k$ and $l$ are constant.

  • If $A=\left[a_{j j}\right]_{m \beta n}$ and $B=\left[b_{i j}\right]_{N \times p}$, then $A B=C-\left[C_{j k}\right]_{w \times p}$, where $c_{j k}=\sum_{j=1}^n a_j b_{j k}$

(i) $A(B C)=(A B) C$

(ii) $A(B+C)=A B+A C$

(iii) $(A+B) C=A C+B C$

  •  If $A=\left[a_W\right]_{n \times N}$, then $A$ or $A^T=\left[a_N\right]_{N \times m}$

(i) $\left(A^{\prime}\right)^{\prime}=A$,

(ii) $(k t)^{\prime}=k A^{\prime}$,

(iii) $(A+B)^{\prime}=A^{\prime}+B^{\prime}$;

(iv) $(A B)^{\prime}=B \cdot A^{\prime}$

  • $A$ is a symmetric matrix if $A^{\prime}=A$.

  • A is a skew symmetric matrix if $A^{\prime}=-A^{\prime}$.

  • Any aquare matrix can be represented as the sum of a symmetric and a skew symmetric matrix.

  • Elementary operations of a matrix are as follows:

(i) $R_i \leftrightarrow R_j$ or $C_i \leftrightarrow C_i$

(ii) $R_l \rightarrow k R_i$ or $C_l \rightarrow k C_i$

(iii) $R_i \rightarrow R_i+k R_j$ or $C_i \rightarrow C_i+k C_j$

  • If $A$ and $B$ re two square matrices such that $A B-B A=I$, then $B$ is the inverse matrix of $A$ and is denoted by $A^{-1}$ and $A$ is the inverse of $\mathrm{B}$.

Competitive Exams after 12th Science
More Free Study Material for Matrices
icons
Revision notes
744k views 12k downloads
icons
Important questions
669.9k views 11k downloads
icons
Ncert books
681.3k views 10k downloads

Practice and Master the Concepts of Matrices with NCERT Solutions Exercises for Class 12 Maths Chapter 3

Exercise 3.1: This exercise introduces the concept of matrices and their notation. Students will learn about different types of matrices, including square matrices, symmetric matrices, and skew-symmetric matrices. They will also practice performing various operations on matrices, such as addition, subtraction, and multiplication.

Exercise 3.2: In this exercise, students will learn about the properties of matrices and practice using these properties to solve problems. They will learn about the commutative, associative, and distributive properties of matrices, as well as the properties of the identity matrix and the zero matrix.

Exercise 3.3: This exercise focuses on matrix multiplication and its properties. Students will learn about the associative property, the distributive property, and the existence of identity matrices and inverse matrices. They will also practice finding the inverse of a matrix and solving equations involving matrices.

Exercise 3.4: In this exercise, students will learn about the transpose of a matrix and its properties. They will also practice finding the transpose of a given matrix and using the properties of the transpose to solve problems.

Miscellaneous Exercise: This exercise includes a mix of questions covering all the concepts taught in the chapter. Students will have to apply their knowledge of matrices to solve various problems and answer questions. They will also practice finding the inverse of a matrix, solving equations involving matrices, and using matrix properties to solve problems.


Overall, this chapter is an important topic in linear algebra and covers the fundamental concepts of matrices, including matrix notation, matrix operations, matrix properties, and matrix multiplication.


NCERT Solutions Class 12 Maths Chapter 3 Exercises

Chapter 3 - Matrices Exercises in PDF Format

Exercise 3.1 : Types of Matrices

10 Questions (5 Short Answers, 5 Long Answers).

Exercise 3.2 : Operations on Matrices

22 Questions (3 Short Answers, 19 Long Answers).

Exercise 3.3 : Transpose of Matrices

12 Questions (4 Short Answers, 8 Long Answers).

Exercise 3.4 : Inverse of Matrices

18 Questions (18 Short Answers). (Not in current syllabus)


Mastering Class 12 Maths Chapter 3: Matrices - MCQs, Question and Answers, and Tips for Success

Exercise 3.1

1. In the matrix \[\mathbf{A=\left[ \begin{matrix} 2 & 5 & 19 & -7  \\ 35 & -2 & \dfrac{5}{2} & 12  \\ \sqrt{3} & 1 & -5 & 17  \\ \end{matrix} \right]}\], write

i. The order of the matrix.

Ans: The order of a matrix is \[m\times n\] where \[m\] is the number of rows and \[n\] is the number of columns. Therefore, here the order is \[3\times 4\].

ii. The number of elements.

Ans: Since the order of the given matrix is \[3\times 4\] therefore, the number of elements in it is \[3\times 4=12\].

iii. Write the elements \[\mathbf{{{a}_{13}},{{a}_{21}},{{a}_{33}},{{a}_{24}},{{a}_{23}}}\]

Ans: The elements are given as \[{{a}_{mn}}\] . Therefore, here \[{{a}_{13}}=19\] , \[{{a}_{21}}=35\] , \[{{a}_{33}}=-5\] , \[{{a}_{24}}=12\] , \[{{a}_{23}}=\dfrac{5}{2}\].


2. If a matrix has \[24\] elements, what are the possible order it can have? What if it has \[13\] elements?

Ans: The order of a matrix is \[m\times n\] where \[m\] is the number of rows and \[n\] is the number of columns. To find the possible orders of a matrix, we have to find all the ordered pairs of natural numbers whose product is \[24\] .

\[\therefore \left( 1\times 24 \right),\left( 24\times 1 \right),\left( 2\times 12 \right),\left( 12\times 2 \right),\left( 3\times 8 \right),\left( 8\times 3 \right),\left( 4\times 6 \right),\left( 6\times 4 \right)\] are all the possible ordered pairs here.

If the matrix had \[13\] elements, then the ordered pairs would be \[\left( 1\times 13 \right)\] and \[\left( 13\times 1 \right)\].


3. If a matrix has \[18\] elements, what are the possible orders it can have? What if it has \[5\] elements?

Ans: The order of a matrix is \[m\times n\] where \[m\] is the number of rows and \[n\] is the number of columns. To find the possible orders of a matrix, we have to find all the ordered pairs of natural numbers whose product is \[18\] .

\[\therefore \left( 1\times 18 \right),\left( 18\times 1 \right),\left( 2\times 9 \right),\left( 9\times 2 \right),\left( 3\times 6 \right),\left( 6\times 3 \right)\] are all the possible ordered pairs here.

If the matrix had \[5\] elements, then the ordered pairs would be \[\left( 1\times 5 \right)\] and \[\left( 5\times 1 \right)\].


4. Construct a \[3\times 4\] matrix, whose elements are given by 

i. \[\mathbf{{{a}_{ij}}=\dfrac{1}{2}\left| -3i+j \right|}\]

Ans: Given that \[{{a}_{ij}}=\dfrac{1}{2}\left| -3i+j \right|\] ,

\[\therefore {{a}_{11}}=\dfrac{1}{2}\left| -3\times 1+1 \right|=1\]

\[{{a}_{21}}=\dfrac{1}{2}\left| -3\times 2+1 \right|=\dfrac{5}{2}\]

\[{{a}_{31}}=\dfrac{1}{2}\left| -3\times 3+1 \right|=4\]

\[{{a}_{12}}=\dfrac{1}{2}\left| -3\times 1+2 \right|=\dfrac{1}{2}\]

\[{{a}_{22}}=\dfrac{1}{2}\left| -3\times 2+2 \right|=2\]

\[{{a}_{32}}=\dfrac{1}{2}\left| -3\times 3+2 \right|=\dfrac{7}{2}\]

\[{{a}_{13}}=\dfrac{1}{2}\left| -3\times 1+3 \right|=0\]

\[{{a}_{23}}=\dfrac{1}{2}\left| -3\times 2+3 \right|=\dfrac{3}{2}\]

\[{{a}_{33}}=\dfrac{1}{2}\left| -3\times 3+3 \right|=3\]

\[{{a}_{14}}=\dfrac{1}{2}\left| -3\times 1+4 \right|=\dfrac{1}{2}\]

\[{{a}_{24}}=\dfrac{1}{2}\left| -3\times 2+4 \right|=1\]

\[{{a}_{34}}=\dfrac{1}{2}\left| -3\times 3+4 \right|=\dfrac{5}{2}\]

Thus, the required matrix is \[A=\left[ \begin{matrix} 1 & \dfrac{1}{2} & 0 & \dfrac{1}{2}  \\ \dfrac{5}{2} & 2 & \dfrac{3}{2} & 1  \\ 4 & \dfrac{7}{2} & 3 & \dfrac{5}{2}  \\ \end{matrix} \right]\].

ii. \[\mathbf{{{a}_{ij}}=2i-j}\]

Ans: A \[3\times 4\] matrix is given by \[A=\left[ \begin{matrix} {{a}_{11}} & {{a}_{12}} & {{a}_{13}} & {{a}_{14}}  \\ {{a}_{21}} & {{a}_{22}} & {{a}_{23}} & {{a}_{24}}  \\ {{a}_{31}} & {{a}_{32}} & {{a}_{33}} & {{a}_{34}}  \\ \end{matrix} \right]\]

Given that \[{{a}_{ij}}=2i-j\] ,

\[\therefore {{a}_{11}}=2\times 1-1=1\]

\[{{a}_{21}}=2\times 2-1=3\]

\[{{a}_{31}}=2\times 3-1=5\]

\[{{a}_{12}}=2\times 1-2=0\]

\[{{a}_{22}}=2\times 2-2=4\]

\[{{a}_{32}}=2\times 3-2=4\]

\[{{a}_{13}}=2\times 1-3=-1\]

\[{{a}_{23}}=2\times 2-3=1\]

\[{{a}_{33}}=2\times 3-3=3\]

\[{{a}_{14}}=2\times 1-4=-2\]

\[{{a}_{24}}=2\times 2-4=0\]

\[{{a}_{34}}=2\times 3-4=2\]

Thus, the required matrix is \[A=\left[ \begin{matrix} 1 & 0 & -1 & -2  \\ 3 & 2 & 1 & 0  \\ 5 & 4 & 3 & 2  \\ \end{matrix} \right]\].


5. Find the value of \[x,y,z\] from the following equation:

i. \[\mathbf{\left[ \begin{matrix} 4 & 3  \\ x & 5  \\ \end{matrix} \right]=\left[ \begin{matrix} y & z  \\  1 & 5  \\ \end{matrix} \right]}\]

Ans: Given \[\left[ \begin{matrix} 4 & 3  \\  x & 5  \\ \end{matrix} \right]=\left[ \begin{matrix} y & z  \\ 1 & 5  \\ \end{matrix} \right]\] 

Comparing the corresponding elements we get,

\[x=1,y=4,z=3\]

ii. \[\mathbf{\left[ \begin{matrix} x+y & 2  \\ 5+z & xy  \\ \end{matrix} \right]=\left[ \begin{matrix} 6 & 2  \\ 5 & 8  \\ \end{matrix} \right]}\]

Ans: Given \[\left[ \begin{matrix} x+y & 2  \\ 5+z & xy  \\ \end{matrix} \right]=\left[ \begin{matrix} 6 & 2  \\ 5 & 8  \\ \end{matrix} \right]\]

Comparing the corresponding elements we get,

\[x+y=6,xy=8,5+z=5\]

Now, \[\because 5+z=5\]

\[\Rightarrow z=0\]

We know that, \[{{\left( x-y \right)}^{2}}={{\left( x+y \right)}^{2}}-4xy\]

\[\Rightarrow {{\left( x-y \right)}^{2}}=36-32\]

\[\Rightarrow \left( x-y \right)=\pm 2\]

When \[\left( x-y \right)=2\] and \[\left( x+y \right)=6\],

We get \[x=4,y=2\]

When \[\left( x-y \right)=-2\] and \[\left( x+y \right)=6\],

We get \[x=2,y=4\]

\[\therefore x=4,y=2,z=0\] or \[\therefore x=2,y=4,z=0\]

iii. \[\mathbf{\left[ \begin{matrix} x+y+z  \\ x+z  \\ y+z  \\ \end{matrix} \right]=\left[ \begin{matrix} 9  \\ 5  \\ 7  \\ \end{matrix} \right]}\]

Ans: Given \[\left[ \begin{matrix} x+y+z  \\ x+z  \\ y+z  \\ \end{matrix} \right]=\left[ \begin{matrix} 9  \\ 5  \\ 7  \\ \end{matrix} \right]\]

Comparing the corresponding elements we get,

\[x+y+z=9\]                      …(1)

\[x+z=5\]                              …(2)

\[y+z=7\]                              …(3)

From equation (1) and (2),

\[y+5=9\]

\[\Rightarrow y=4\]

From equation (3) we have,

\[4+z=7\]

\[\Rightarrow z=3\]

\[x+z=5\]

\[\Rightarrow x=2\]

\[\therefore x=2,y=4,z=3\]


6. Find the value of \[\mathbf{a,b,c,d}\] from the equation:

\[\mathbf{\left[ \begin{matrix} a-b & 2a+c  \\ 2a-b & 3c+d  \\ \end{matrix} \right]=\left[ \begin{matrix} -1 & 5  \\ 0 & 13  \\ \end{matrix} \right]}\]

Ans: Given \[\left[ \begin{matrix} a-b & 2a+c  \\ 2a-b & 3c+d  \\ \end{matrix} \right]=\left[ \begin{matrix} -1 & 5  \\ 0 & 13  \\ \end{matrix} \right]\]

Comparing the corresponding elements we get,

\[a-b=-1\]                              …(1)

\[2a-b=0\]                              …(2)

\[2a+c=5\]                              …(3)

\[3c+d=13\]                                        …(4)

From equation (2),

\[b=2a\]

From equation (1),

\[a-2a=-1\]

\[\Rightarrow a=1\]

\[\Rightarrow b=2\]

From equation (3),

\[2\times 1+c=5\]

\[\Rightarrow c=3\]

From equation (4),

\[3\times 3+d=13\]

\[\Rightarrow d=4\]

\[\therefore a=1,b=2,c=3,d=4\]


7. \[\mathbf{A={{\left[ {{a}_{y}} \right]}_{m\times n}}}\] is a square matrix, if

  1. m<n

  2. m>n

  3. m=n

  4. None of these

Ans: A given matrix is said to be a square matrix if the number of rows is equal to the number of columns.

\[\therefore A={{\left[ {{a}_{y}} \right]}_{m\times n}}\] is a square matrix if, \[m=n\].

Thus, option (C) is correct.


8. Which of the given values of \[x\] and \[y\] make the following pair of matrices equal \[\mathbf{\left[ \begin{matrix} 3x+7 & 5  \\ y+1 & 2-3x  \\ \end{matrix} \right]=\left[ \begin{matrix} 0 & y-2  \\  8 & 4  \\ \end{matrix} \right]}\]

  1. \[\mathbf{x=\dfrac{-1}{3},y=7}\]

  2. Not possible to find

  3. \[\mathbf{y=7,x=\dfrac{-2}{3}}\]

  4. \[\mathbf{x=\dfrac{-1}{3},y=\dfrac{-2}{3}}\]

Ans: Given \[\left[ \begin{matrix} 3x+7 & 5  \\ y+1 & 2-3x  \\ \end{matrix} \right]=\left[ \begin{matrix} 0 & y-2  \\  8 & 4  \\ \end{matrix} \right]\]

Comparing the corresponding elements we get,

\[3x+7=0\]

\[\Rightarrow x=-\dfrac{7}{3}\]

\[y-2=5\]

\[\Rightarrow y=7\]

\[y+1=8\]

\[\Rightarrow y=7\]

\[2-3x=4\]

\[\Rightarrow x=-\dfrac{2}{3}\]

Since we get two different values of \[x\] ,which is not possible. It is not possible to find the values of \[x\] and \[y\] for which the given matrices are equal.

Thus, the correct option is (B).


9. The number of all possible matrices of order \[\mathbf{3\times 3}\] with each entry \[0\] or \[1\] is:

  1. \[\mathbf{27}\]

  2. \[\mathbf{18}\]

  3. \[\mathbf{81}\]

  4. \[\mathbf{512}\]

Ans: Given a matrix of the order \[3\times 3\] has nine elements and each of these elements can be either \[0\] or \[1\] .

Now, each of the nine elements can be filled in two possible ways.

Therefore, the required number of possible matrices is \[{{2}^{9}}=512\].


Exercise 3.2

1. Let \[\mathbf{A=\left[ \begin{matrix} 2 & 4  \\ 3 & 2  \\ \end{matrix} \right],B=\left[ \begin{matrix} 1 & 3  \\ -2 & 5  \\ \end{matrix} \right],C=\left[ \begin{matrix} -2 & 5  \\ 3 & 4  \\ \end{matrix} \right]}\]

Find each of the following

i. \[\mathbf{A+B}\]

Ans: Given \[A=\left[ \begin{matrix}  2 & 4  \\  3 & 2  \\ \end{matrix} \right],B=\left[ \begin{matrix}  1 & 3  \\ -2 & 5  \\ \end{matrix} \right]\]

\[\therefore A+B=\left[ \begin{matrix}  2 & 4  \\  3 & 2  \\ \end{matrix} \right]+\left[ \begin{matrix} 1 & 3  \\ -2 & 5  \\ \end{matrix} \right]\]

\[\Rightarrow A+B=\left[ \begin{matrix} 2+1 & 4+3  \\ 3-2 & 2+5  \\ \end{matrix} \right]\]

\[\Rightarrow A+B=\left[ \begin{matrix}  3 & 7  \\ 1 & 7  \\ \end{matrix} \right]\]

ii. \[\mathbf{A-B}\]

Ans: Given \[A=\left[ \begin{matrix} 2 & 4  \\ 3 & 2  \\ \end{matrix} \right],B=\left[ \begin{matrix} 1 & 3  \\ -2 & 5  \\ \end{matrix} \right]\]

\[\therefore A-B=\left[ \begin{matrix}  2 & 4  \\ 3 & 2  \\ \end{matrix} \right]-\left[ \begin{matrix} 1 & 3  \\ -2 & 5  \\ \end{matrix} \right]\]

\[\Rightarrow A-B=\left[ \begin{matrix} 2-1 & 4-3  \\ 3+2 & 2-5  \\ \end{matrix} \right]\]

\[\Rightarrow A-B=\left[ \begin{matrix} 1 & 1  \\ 5 & -3  \\ \end{matrix} \right]\]

iii. \[\mathbf{3A-C}\]

Ans: Given \[A=\left[ \begin{matrix} 2 & 4  \\ 3 & 2  \\ \end{matrix} \right],C=\left[ \begin{matrix} -2 & 5  \\ 3 & 4  \\ \end{matrix} \right]\]

\[\therefore 3A-C=3\left[ \begin{matrix} 2 & 4  \\ 3 & 2  \\ \end{matrix} \right]-\left[ \begin{matrix} -2 & 5  \\ 3 & 4  \\ \end{matrix} \right]\]

\[\Rightarrow 3A-C=\left[ \begin{matrix} 6 & 12  \\ 9 & 6  \\ \end{matrix} \right]-\left[ \begin{matrix} -2 & 5  \\ 3 & 4  \\ \end{matrix} \right]\]

\[\Rightarrow 3A-C=\left[ \begin{matrix} 6+2 & 12-5  \\ 9-3 & 6-4  \\ \end{matrix} \right]\]

\[\Rightarrow 3A-C=\left[ \begin{matrix} 8 & 7  \\ 6 & 2  \\ \end{matrix} \right]\]

iv. \[\mathbf{AB}\]

Ans: Given \[A=\left[ \begin{matrix} 2 & 4  \\ 3 & 2  \\ \end{matrix} \right],B=\left[ \begin{matrix} 1 & 3  \\ -2 & 5  \\ \end{matrix} \right]\]

\[\therefore AB=\left[ \begin{matrix} 2 & 4  \\ 3 & 2  \\ \end{matrix} \right]\left[ \begin{matrix} 1 & 3  \\ -2 & 5  \\ \end{matrix} \right]\]

\[\Rightarrow AB=\left[ \begin{matrix} 2\left( 1 \right)+4\left( -2 \right) & 2\left( 3 \right)+4\left( 5 \right)  \\ 3\left( 1 \right)+2\left( -2 \right) & 3\left( 3 \right)+2\left( 5 \right)  \\ \end{matrix} \right]\]

\[\Rightarrow AB=\left[ \begin{matrix}  -6 & 26  \\ -1 & 19  \\ \end{matrix} \right]\]

v. \[\mathbf{BA}\]

Ans: Given \[A=\left[ \begin{matrix} 2 & 4  \\ 3 & 2  \\ \end{matrix} \right],B=\left[ \begin{matrix} 1 & 3  \\ -2 & 5  \\ \end{matrix} \right]\]

\[\therefore BA=\left[ \begin{matrix} 1 & 3  \\ -2 & 5  \\ \end{matrix} \right]\left[ \begin{matrix} 2 & 4  \\ 3 & 2  \\ \end{matrix} \right]\]

\[\Rightarrow BA=\left[ \begin{matrix} 1\left( 2 \right)+3\left( 3 \right) & 1\left( 4 \right)+3\left( 2 \right)  \\ -2\left( 2 \right)+5\left( 3 \right) & -2\left( 4 \right)+5\left( 2 \right)  \\ \end{matrix} \right]\]

\[\Rightarrow BA=\left[ \begin{matrix} 11 & 10  \\ 11 & 2  \\ \end{matrix} \right]\]


2. Compute the following:

i. \[\mathbf{\left[ \begin{matrix} a & b  \\ -b & a  \\ \end{matrix} \right]+\left[ \begin{matrix} a & b  \\ b & a  \\ \end{matrix} \right]}\]

Ans: We have to find \[\left[ \begin{matrix} a & b  \\ -b & a  \\ \end{matrix} \right]+\left[ \begin{matrix} a & b  \\ b & a  \\ \end{matrix} \right]\]

\[\therefore \left[ \begin{matrix} a & b  \\ -b & a  \\ \end{matrix} \right]+\left[ \begin{matrix} a & b  \\ b & a  \\ \end{matrix} \right]=\left[ \begin{matrix} a+a & b+b  \\ -b+b & a+a  \\ \end{matrix} \right]\]

\[\Rightarrow \left[ \begin{matrix} a & b  \\ -b & a  \\ \end{matrix} \right]+\left[ \begin{matrix} a & b  \\ b & a  \\ \end{matrix} \right]=\left[ \begin{matrix} 2a & 2b  \\ 0 & 2a  \\ \end{matrix} \right]\]

ii. \[\mathbf{\left[ \begin{matrix}  {{a}^{2}}+{{b}^{2}} & {{b}^{2}}+{{c}^{2}}  \\ {{a}^{2}}+{{c}^{2}} & {{a}^{2}}+{{b}^{2}}  \\ \end{matrix} \right]+\left[ \begin{matrix} 2ab & 2bc  \\ -2ac & -2ab  \\ \end{matrix} \right]}\]

Ans: We have to find 

\[\left[ \begin{matrix}  {{a}^{2}}+{{b}^{2}} & {{b}^{2}}+{{c}^{2}}  \\ {{a}^{2}}+{{c}^{2}} & {{a}^{2}}+{{b}^{2}}  \\ \end{matrix} \right]+\left[ \begin{matrix} 2ab & 2bc  \\ -2ac & -2ab  \\ \end{matrix} \right]\]

\[\therefore \left[ \begin{matrix} {{a}^{2}}+{{b}^{2}} & {{b}^{2}}+{{c}^{2}}  \\ {{a}^{2}}+{{c}^{2}} & {{a}^{2}}+{{b}^{2}}  \\ \end{matrix} \right]+\left[ \begin{matrix} 2ab & 2bc  \\ -2ac & -2ab  \\ \end{matrix} \right]=\left[ \begin{matrix} {{a}^{2}}+{{b}^{2}}+2ab & {{b}^{2}}+{{c}^{2}}+2bc  \\ {{a}^{2}}+{{c}^{2}}-2ac & {{a}^{2}}+{{b}^{2}}-2ab  \\ \end{matrix} \right]\]

\[\Rightarrow \left[ \begin{matrix} {{a}^{2}}+{{b}^{2}} & {{b}^{2}}+{{c}^{2}}  \\ {{a}^{2}}+{{c}^{2}} & {{a}^{2}}+{{b}^{2}}  \\ \end{matrix} \right]+\left[ \begin{matrix} 2ab & 2bc  \\ -2ac & -2ab  \\ \end{matrix} \right]=\left[ \begin{matrix} {{\left( a+b \right)}^{2}} & {{\left( b+c \right)}^{2}}  \\ {{\left( a-c \right)}^{2}} & {{\left( a-b \right)}^{2}}  \\ \end{matrix} \right]\]

iii. \[\left[ \begin{matrix} -1 & 4 & -6  \\ 8 & 5 & 16  \\ 2 & 8 & 5  \\ \end{matrix} \right]+\left[ \begin{matrix} 12 & 7 & 6  \\ 8 & 0 & 5  \\ 3 & 2 & 4  \\ \end{matrix} \right]\]

Ans: We have to find \[\left[ \begin{matrix} -1 & 4 & -6  \\ 8 & 5 & 16  \\ 2 & 8 & 5  \\ \end{matrix} \right]+\left[ \begin{matrix} 12 & 7 & 6  \\ 8 & 0 & 5  \\ 3 & 2 & 4  \\ \end{matrix} \right]\]

\[\therefore \left[ \begin{matrix} -1 & 4 & -6  \\ 8 & 5 & 16  \\ 2 & 8 & 5  \\ \end{matrix} \right]+\left[ \begin{matrix} 12 & 7 & 6  \\ 8 & 0 & 5  \\ 3 & 2 & 4  \\ \end{matrix} \right]=\left[ \begin{matrix} -1+12 & 4+7 & -6+6  \\ 8+8 & 5+0 & 16+5  \\ 2+3 & 8+2 & 5+4  \\ \end{matrix} \right]\]

\[\Rightarrow \left[ \begin{matrix} -1 & 4 & -6  \\ 8 & 5 & 16  \\ 2 & 8 & 5  \\ \end{matrix} \right]+\left[ \begin{matrix} 12 & 7 & 6  \\ 8 & 0 & 5  \\ 3 & 2 & 4  \\ \end{matrix} \right]=\left[ \begin{matrix} 11 & 11 & 0  \\ 16 & 5 & 21  \\ 5 & 10 & 9  \\ \end{matrix} \right]\]

iv. \[\left[ \begin{matrix} {{\cos }^{2}}x & {{\sin }^{2}}x  \\ {{\sin }^{2}}x & {{\cos }^{2}}x  \\ \end{matrix} \right]+\left[ \begin{matrix} {{\sin }^{2}}x & {{\cos }^{2}}x  \\ {{\cos }^{2}}x & {{\sin }^{2}}x  \\ \end{matrix} \right]\]

Ans: We have to find \[\left[ \begin{matrix} {{\cos }^{2}}x & {{\sin }^{2}}x  \\   {{\sin }^{2}}x & {{\cos }^{2}}x  \\ \end{matrix} \right]+\left[ \begin{matrix}   {{\sin }^{2}}x & {{\cos }^{2}}x  \\ {{\cos }^{2}}x & {{\sin }^{2}}x  \\ \end{matrix} \right]\]

\[\therefore \left[ \begin{matrix} {{\cos }^{2}}x & {{\sin }^{2}}x  \\ {{\sin }^{2}}x & {{\cos }^{2}}x  \\ \end{matrix} \right]+\left[ \begin{matrix} {{\sin }^{2}}x & {{\cos }^{2}}x  \\ {{\cos }^{2}}x & {{\sin }^{2}}x  \\ \end{matrix} \right]=\left[ \begin{matrix} {{\cos }^{2}}x+{{\sin }^{2}}x & {{\sin }^{2}}x+{{\cos }^{2}}x  \\ {{\sin }^{2}}x+{{\cos }^{2}}x & {{\cos }^{2}}x+{{\sin }^{2}}x  \\ \end{matrix} \right]\]

\[\Rightarrow \left[ \begin{matrix} {{\cos }^{2}}x & {{\sin }^{2}}x  \\ {{\sin }^{2}}x & {{\cos }^{2}}x  \\ \end{matrix} \right]+\left[ \begin{matrix} {{\sin }^{2}}x & {{\cos }^{2}}x  \\ {{\cos }^{2}}x & {{\sin }^{2}}x  \\ \end{matrix} \right]=\left[ \begin{matrix} 1 & 1  \\ 1 & 1  \\ \end{matrix} \right]\]


3. Compute the indicated products

i. \[\mathbf{\left[ \begin{matrix} a & b  \\ -b & a  \\ \end{matrix} \right]\left[ \begin{matrix} a & -b  \\ b & a  \\ \end{matrix} \right]}\]

Ans: We have to find \[\left[ \begin{matrix} a & b  \\ -b & a  \\ \end{matrix} \right]\left[ \begin{matrix} a & -b  \\ b & a  \\ \end{matrix} \right]\]

\[\therefore \left[ \begin{matrix} a & b  \\ -b & a  \\ \end{matrix} \right]\left[ \begin{matrix} a & -b  \\ b & a  \\ \end{matrix} \right]=\left[ \begin{matrix} a\left( a \right)+b\left( b \right) & a\left( -b \right)+b\left( a \right)  \\ -b\left( a \right)+a\left( b \right) & -b\left( -b \right)+a\left( a \right)  \\ \end{matrix} \right]\]

\[\Rightarrow \left[ \begin{matrix} a & b  \\ -b & a  \\ \end{matrix} \right]\left[ \begin{matrix} a & -b  \\ b & a  \\ \end{matrix} \right]=\left[ \begin{matrix}   {{a}^{2}}+{{b}^{2}} & 0  \\  0 & {{a}^{2}}+{{b}^{2}}  \\ \end{matrix} \right]\]

ii. \[\mathbf{\left[ \begin{matrix} 1  \\   2  \\   3  \\ \end{matrix} \right]\left[ \begin{matrix}   2 & 3 & 4  \\ \end{matrix} \right]}\]

Ans: We have to find \[\left[ \begin{matrix}    1  \\   2  \\   3  \\ \end{matrix} \right]\left[ \begin{matrix}  2 & 3 & 4  \\ \end{matrix} \right]\]

\[\therefore \left[ \begin{matrix}    1  \\   2  \\   3  \\ \end{matrix} \right]\left[ \begin{matrix}   2 & 3 & 4  \\ \end{matrix} \right]=\left[ \begin{matrix}   1\left( 2 \right) & 1\left( 3 \right) & 1\left( 4 \right)  \\   2\left( 2 \right) & 2\left( 3 \right) & 2\left( 4 \right)  \\   3\left( 2 \right) & 3\left( 3 \right) & 3\left( 4 \right)  \\ \end{matrix} \right]\]

\[\Rightarrow \left[ \begin{matrix}   1  \\   2  \\   3  \\ \end{matrix} \right]\left[ \begin{matrix}   2 & 3 & 4  \\ \end{matrix} \right]=\left[ \begin{matrix}   2 & 3 & 4  \\   4 & 6 & 8  \\   6 & 9 & 12  \\ \end{matrix} \right]\]

iii. \[\mathbf{\left[ \begin{matrix}   1 & -2  \\  2 & 3  \\ \end{matrix} \right]\left[ \begin{matrix}   1 & 2 & 3  \\   2 & 3 & 1  \\ \end{matrix} \right]}\]

Ans: We have to find \[\left[ \begin{matrix}   1 & -2  \\   2 & 3  \\\end{matrix} \right]\left[ \begin{matrix}   1 & 2 & 3  \\   2 & 3 & 1  \\ \end{matrix} \right]\]

\[\therefore \left[ \begin{matrix}   1 & -2  \\   2 & 3  \\ \end{matrix} \right]\left[ \begin{matrix}   1 & 2 & 3  \\   2 & 3 & 1  \\ \end{matrix} \right]=\left[ \begin{matrix}   1\left( 1 \right)-2\left( 2 \right) & 1\left( 2 \right)-2\left( 3 \right) & 1\left( 3 \right)-2\left( 1 \right)  \\   2\left( 1 \right)+3\left( 2 \right) & 2\left( 2 \right)+3\left( 3 \right) & 2\left( 3 \right)+3\left( 1 \right)  \\ \end{matrix} \right]\]

\[\Rightarrow \left[ \begin{matrix}   1 & -2  \\   2 & 3  \\ \end{matrix} \right]\left[ \begin{matrix}   1 & 2 & 3  \\   2 & 3 & 1  \\ \end{matrix} \right]=\left[ \begin{matrix}   -3 & -4 & 1  \\   8 & 13 & 9  \\ \end{matrix} \right]\]

iv. \[\mathbf{\left[ \begin{matrix}  2 & 3 & 4  \\   3 & 4 & 5  \\   4 & 5 & 6  \\ \end{matrix} \right]\left[ \begin{matrix}    1 & -3 & 5  \\   0 & 2 & 4  \\   3 & 0 & 5  \\ \end{matrix} \right]}\]

Ans: We have to find \[\left[ \begin{matrix}  2 & 3 & 4  \\   3 & 4 & 5  \\   4 & 5 & 6  \\ \end{matrix} \right]\left[ \begin{matrix}    1 & -3 & 5  \\   0 & 2 & 4  \\   3 & 0 & 5  \\ \end{matrix} \right]\]

\[\therefore \left[ \begin{matrix}   2 & 3 & 4  \\   3 & 4 & 5  \\   4 & 5 & 6  \\ \end{matrix} \right]\left[ \begin{matrix}  1 & -3 & 5  \\   0 & 2 & 4  \\   3 & 0 & 5  \\ \end{matrix} \right]=\left[ \begin{matrix}   2\left( 1 \right)+3\left( 0 \right)+4\left( 3 \right) & 2\left( -3 \right)+3\left( 2 \right)+4\left( 0 \right) & 2\left( 5 \right)+3\left( 4 \right)+4\left( 5 \right)  \\   3\left( 1 \right)+4\left( 0 \right)+5\left( 3 \right) & 3\left( -3 \right)+4\left( 2 \right)+5\left( 0 \right) & 3\left( 5 \right)+4\left( 4 \right)+5\left( 5 \right)  \\   4\left( 1 \right)+5\left( 0 \right)+5\left( 3 \right) & 4\left( -3 \right)+5\left( 2 \right)+6\left( 0 \right) & 4\left( 5 \right)+5\left( 4 \right)+6\left( 5 \right)  \\ \end{matrix} \right]\]

\[\Rightarrow \left[ \begin{matrix}   2 & 3 & 4  \\   3 & 4 & 5  \\   4 & 5 & 6  \\ \end{matrix} \right]\left[ \begin{matrix}   1 & -3 & 5  \\   0 & 2 & 4  \\   3 & 0 & 5  \\ \end{matrix} \right]=\left[ \begin{matrix}   14 & 0 & 42  \\  18 & -1 & 56  \\   22 & -2 & 70  \\ \end{matrix} \right]\]

v. \[\mathbf{\left[ \begin{matrix}   2 & 1  \\   3 & 2  \\   -1 & 1  \\ \end{matrix} \right]\left[ \begin{matrix}   1 & 0 & 1  \\   -1 & 2 & 1  \\ \end{matrix} \right]}\]

Ans: We have to find \[\left[ \begin{matrix}   2 & 1  \\   3 & 2  \\   -1 & 1  \\ \end{matrix} \right]\left[ \begin{matrix}   1 & 0 & 1  \\   -1 & 2 & 1  \\ \end{matrix} \right]\]

\[\therefore \left[ \begin{matrix}   2 & 1  \\   3 & 2  \\   -1 & 1  \\ \end{matrix} \right]\left[ \begin{matrix}   1 & 0 & 1  \\   -1 & 2 & 1  \\ \end{matrix} \right]=\left[ \begin{matrix}  2\left( 1 \right)+1\left( -1 \right) & 2\left( 0 \right)+1\left( 2 \right) & 2\left( 1 \right)+1\left( 1 \right)  \\   3\left( 1 \right)+2\left( -1 \right) & 3\left( 0 \right)+2\left( 2 \right) & 3\left( 1 \right)+2\left( 1 \right)  \\   -1\left( 1 \right)+1\left( -1 \right) & -1\left( 0 \right)+1\left( 2 \right) & -1\left( 1 \right)+1\left( 1 \right)  \\ \end{matrix} \right]\]

\[\Rightarrow \left[ \begin{matrix}   2 & 1  \\   3 & 2  \\   -1 & 1  \\ \end{matrix} \right]\left[ \begin{matrix}   1 & 0 & 1  \\   -1 & 2 & 1  \\ \end{matrix} \right]=\left[ \begin{matrix}   1 & 2 & 3  \\   1 & 4 & 5  \\   -2 & 2 & 0  \\ \end{matrix} \right]\]

vi. \[\mathbf{\left[ \begin{matrix}   3 & -1 & 3  \\   -1 & 0 & 2  \\ \end{matrix} \right]\left[ \begin{matrix}   2 & -3  \\   1 & 0  \\   3 & 1  \\ \end{matrix} \right]}\]

Ans: We have to find \[\left[ \begin{matrix}  3 & -1 & 3  \\   -1 & 0 & 2  \\ \end{matrix} \right]\left[ \begin{matrix}   2 & -3  \\   1 & 0  \\   3 & 1  \\ \end{matrix} \right]\]

\[\therefore \left[ \begin{matrix}   3 & -1 & 3  \\   -1 & 0 & 2  \\ \end{matrix} \right]\left[ \begin{matrix}   2 & -3  \\  1 & 0  \\   3 & 1  \\ \end{matrix} \right]=\left[ \begin{matrix}  3\left( 2 \right)-1\left( 1 \right)+3\left( 3 \right) & 3\left( -3 \right)-1\left( 0 \right)+3\left( 1 \right)  \\  -1\left( 2 \right)+0\left( 1 \right)+2\left( 3 \right) & -1\left( -3 \right)+0\left( 0 \right)+2\left( 1 \right)  \\ \end{matrix} \right]\]

\[\Rightarrow \left[ \begin{matrix}   3 & -1 & 3  \\   -1 & 0 & 2  \\ \end{matrix} \right]\left[ \begin{matrix}   2 & -3  \\   1 & 0  \\   3 & 1  \\ \end{matrix} \right]=\left[ \begin{matrix} 14 & -6  \\   4 & 5  \\ \end{matrix} \right]\]


4.If,

\[\mathbf{A=\left[\begin{matrix} 1 & 2 & -3 \\ 5 & 0 & 2 \\ 1 & -1 & 1  \\ \end{matrix}\right],B=\left[ \begin{matrix}  3 & -1 & 2  \\  4 & 2 & 5  \\  2 & 0 & 3  \\ \end{matrix}\right],C=\left[ \begin{matrix}  4 & 1 & 2  \\  0 & 3 & 2  \\  1 & -2 & 3  \\ \end{matrix}\right]}\],

then Compute ( A+B ) and ( B-C ).Also, verify that A+( B-C)=( A+B )-C}.

Ans: Given  \[A=\left[ \begin{matrix}   1 & 2 & -3  \\   5 & 0 & 2  \\   1 & -1 & 1  \\ \end{matrix} \right],B=\left[ \begin{matrix}   3 & -1 & 2  \\   4 & 2 & 5  \\   2 & 0 & 3  \\ \end{matrix} \right],C=\left[ \begin{matrix}   4 & 1 & 2  \\   0 & 3 & 2  \\   1 & -2 & 3  \\ \end{matrix} \right]\]

\[\therefore A+B=\left[ \begin{matrix}   1 & 2 & -3  \\   5 & 0 & 2  \\   1 & -1 & 1  \\ \end{matrix} \right]+\left[ \begin{matrix}   3 & -1 & 2  \\   4 & 2 & 5  \\   2 & 0 & 3  \\ \end{matrix} \right]\]

\[\Rightarrow A+B=\left[ \begin{matrix}   1+3 & 2-1 & -3+2  \\   5+4 & 0+2 & 2+5 \\  1+2 & -1+0 & 1+3  \\ \end{matrix} \right]\]

\[\Rightarrow A+B=\left[ \begin{matrix}   4 & 1 & -1  \\   9 & 2 & 7  \\   3 & -1 & 4  \\ \end{matrix} \right]\]

And \[B-C=\left[ \begin{matrix}   3 & -1 & 2  \\   4 & 2 & 5  \\   2 & 0 & 3  \\ \end{matrix} \right]-\left[ \begin{matrix}   4 & 1 & 2  \\   0 & 3 & 2  \\   1 & -2 & 3  \\ \end{matrix} \right]\]

\[\Rightarrow B-C=\left[ \begin{matrix}   3-4 & -1-1 & 2-2  \\   4-0 & 2-3 & 5-2  \\   2-1 & 0+2 & 3-3  \\ \end{matrix} \right]\]

\[\therefore B-C=\left[ \begin{matrix}   -1 & -2 & 0  \\   4 & -1 & 3  \\   1 & 2 & 0  \\ \end{matrix} \right]\]

Now,

\[A+\left( B-C \right)=\left[ \begin{matrix}  1 & 2 & -3  \\   5 & 0 & 2  \\   1 & -1 & 1  \\ \end{matrix} \right]+\left[ \begin{matrix}   -1 & -2 & 0  \\   4 & -1 & 3  \\   1 & 2 & 0  \\ \end{matrix} \right]\]

\[\Rightarrow A+\left( B-C \right)=\left[ \begin{matrix}   1-1 & 2-2 & -3+0  \\   5+4 & 0-1 & 2+3  \\   1+1 & -1+2 & 1+0  \\ \end{matrix} \right]\]

\[\therefore A+\left( B-C \right)=\left[ \begin{matrix}   0 & 0 & -3  \\   9 & -1 & 5  \\   2 & 1 & 1  \\ \end{matrix} \right]\]                                                  …(1)

And \[\left( A+B \right)-C=\left[ \begin{matrix}   4 & 1 & -1  \\   9 & 2 & 7  \\   3 & -1 & 4  \\ \end{matrix} \right]-\left[ \begin{matrix}  4 & 1 & 2  \\   0 & 3 & 2  \\   1 & -2 & 3  \\ \end{matrix} \right]\]

\[\Rightarrow \left( A+B \right)-C=\left[ \begin{matrix}   4-4 & 1-1 & -1-2  \\   9-0 & 2-3 & 7-2  \\   3-1 & -1+2 & 4-3  \\ \end{matrix} \right]\]

\[\therefore \left( A+B \right)-C=\left[ \begin{matrix}   0 & 0 & -3  \\   9 & -1 & 5  \\   2 & 1 & 1  \\ \end{matrix} \right]\]                                                  …(2)

Thus, from equation (1) and (2),

\[A+\left( B-C \right)=\left( A+B \right)-C\]

Hence proved.


5. If \[A=\left[ \begin{matrix}   \dfrac{2}{3} & 1 & \dfrac{5}{3}  \\   \dfrac{1}{3} & \dfrac{2}{3} & \dfrac{4}{3}  \\   \dfrac{7}{3} & 2 & \dfrac{2}{3}  \\ \end{matrix} \right]\] and \[B=\left[ \begin{matrix}   \dfrac{2}{5} & \dfrac{3}{5} & 1  \\   \dfrac{1}{5} & \dfrac{2}{5} & \dfrac{4}{5}  \\    \dfrac{7}{5} & \dfrac{6}{5} & \dfrac{2}{5}  \\ \end{matrix} \right]\] then compute \[3A-5B\] .

Ans: Given that \[A=\left[ \begin{matrix}   \dfrac{2}{3} & 1 & \dfrac{5}{3}  \\   \dfrac{1}{3} & \dfrac{2}{3} & \dfrac{4}{3}  \\   \dfrac{7}{3} & 2 & \dfrac{2}{3}  \\ \end{matrix} \right]\] and \[B=\left[ \begin{matrix}   \dfrac{2}{5} & \dfrac{3}{5} & 1  \\   \dfrac{1}{5} & \dfrac{2}{5} & \dfrac{4}{5}  \\    \dfrac{7}{5} & \dfrac{6}{5} & \dfrac{2}{5}  \\ \end{matrix} \right]\] 

\[\therefore 3A-5B=3\left[ \begin{matrix}   \dfrac{2}{3} & 1 & \dfrac{5}{3}  \\   \dfrac{1}{3} & \dfrac{2}{3} & \dfrac{4}{3}  \\   \dfrac{7}{3} & 2 & \dfrac{2}{3}  \\ \end{matrix} \right]-5\left[ \begin{matrix}   \dfrac{2}{5} & \dfrac{3}{5} & 1  \\   \dfrac{1}{5} & \dfrac{2}{5} & \dfrac{4}{5}  \\   \dfrac{7}{5} & \dfrac{6}{5} &\dfrac{2}{5}  \\ \end{matrix} \right]\]

\[\Rightarrow 3A-5B=\left[ \begin{matrix}   2 & 3 & 5  \\   1 & 2 & 4  \\   7 & 6 & 2  \\ \end{matrix} \right]-\left[ \begin{matrix}   2 & 3 & 5  \\   1 & 2 & 4  \\   7 & 6 & 2  \\ \end{matrix} \right]\]

\[\Rightarrow 3A-5B=\left[ \begin{matrix}   0 & 0 & 0  \\   0 & 0 & 0  \\   0 & 0 & 0  \\ \end{matrix} \right]\]


6. Simplify \[\mathbf{\cos \theta \left[ \begin{matrix}   \cos \theta  & \sin \theta   \\   -\sin \theta  & \cos \theta   \\ \end{matrix} \right]+\sin \theta \left[ \begin{matrix}  \sin \theta  & -\cos \theta   \\  \cos \theta  & \sin \theta   \\ \end{matrix} \right]}\] .

Ans: We have to simplify \[\cos \theta \left[ \begin{matrix}   \cos \theta  & \sin \theta   \\   -\sin \theta  & \cos \theta   \\ \end{matrix} \right]+\sin \theta \left[ \begin{matrix}  \sin \theta  & -\cos \theta   \\  \cos \theta  & \sin \theta   \\ \end{matrix} \right]\] 

\[\therefore \cos \theta \left[ \begin{matrix} \cos \theta  & \sin \theta   \\   -\sin \theta  & \cos \theta   \\ \end{matrix} \right]+\sin \theta \left[ \begin{matrix}   \sin \theta  & -\cos \theta   \\   \cos \theta  & \sin \theta   \\ \end{matrix} \right]=\left[ \begin{matrix}   {{\cos }^{2}}\theta  & \cos \theta \sin \theta   \\   -\cos \theta \sin \theta  & {{\cos }^{2}}\theta   \\ \end{matrix} \right]+\left[ \begin{matrix}   {{\sin }^{2}}\theta  & -\sin \theta \cos \theta   \\   \sin \theta \cos \theta  & {{\sin }^{2}}\theta   \\ \end{matrix} \right]\]

\[\Rightarrow \cos \theta \left[ \begin{matrix}   \cos \theta  & \sin \theta   \\   -\sin \theta  & \cos \theta   \\ \end{matrix} \right]+\sin \theta \left[ \begin{matrix}   \sin \theta  & -\cos \theta   \\  \cos \theta  & \sin \theta   \\ \end{matrix} \right]=\left[ \begin{matrix}  {{\cos }^{2}}\theta +{{\sin }^{2}}\theta  & \cos \theta \sin \theta -\cos \theta \sin \theta   \\  -\cos \theta \sin \theta +\cos \theta \sin \theta  & {{\cos }^{2}}\theta +{{\sin }^{2}}\theta   \\ \end{matrix} \right]\]

\[\Rightarrow \cos \theta \left[ \begin{matrix}  \cos \theta  & \sin \theta   \\   -\sin \theta  & \cos \theta   \\ \end{matrix} \right]+\sin \theta \left[ \begin{matrix}  \sin \theta  & -\cos \theta   \\  \cos \theta  & \sin \theta   \\ \end{matrix} \right]=\left[ \begin{matrix}   1 & 0  \\   0 & 1  \\ \end{matrix} \right]\]


7. Find \[X\] and \[Y\] ,if

i. \[{X+Y=}\left [ \begin{matrix}   7 & 0  \\   2 & 5  \\ \end{matrix} \right]\] and \[X-Y=\left[ \begin{matrix}   3 & 0  \\   0 & 3  \\ \end{matrix} \right]\]

Ans: Given:

\[X+Y=\left[ \begin{matrix}   7 & 0  \\   2 & 5  \\ \end{matrix} \right]\]        …(1)

\[X-Y=\left[ \begin{matrix}   3 & 0  \\   0 & 3  \\ \end{matrix} \right]\]   …(2)

Adding equations (1) and (2), we get,

\[2X=\left[ \begin{matrix}   7 & 0  \\   2 & 5  \\ \end{matrix} \right]+\left[ \begin{matrix}   3 & 0  \\   0 & 3  \\ \end{matrix} \right]\]

\[\Rightarrow 2X=\left[ \begin{matrix}   10 & 0  \\   2 & 8  \\ \end{matrix} \right]\]

\[\Rightarrow X=\dfrac{1}{2}\left[ \begin{matrix}   10 & 0  \\   2 & 8  \\ \end{matrix} \right]\]

\[\therefore X=\left[ \begin{matrix}   5 & 0  \\   1 & 4  \\ \end{matrix} \right]\]

Now, since \[X+Y=\left[ \begin{matrix}    7 & 0  \\   2 & 5  \\ \end{matrix} \right]\]

\[\Rightarrow \left[ \begin{matrix}  5 & 0  \\   1 & 4  \\ \end{matrix} \right]+Y=\left[ \begin{matrix}   7 & 0  \\   2 & 5  \\ \end{matrix} \right]\]

\[\Rightarrow Y=\left[ \begin{matrix}   7 & 0  \\   2 & 5  \\ \end{matrix} \right]-\left[ \begin{matrix}   5 & 0  \\   1 & 4  \\ \end{matrix} \right]\]

\[\therefore Y=\left[ \begin{matrix}   2 & 0  \\   1 & 1  \\ \end{matrix} \right]\]

\[\therefore X=\left[ \begin{matrix}   5 & 0  \\   1 & 4  \\ \end{matrix} \right],Y=\left[ \begin{matrix}   2 & 0  \\   1 & 1  \\ \end{matrix} \right]\]

ii. \[\mathbf{2X+3Y=\left[ \begin{matrix}   2 & 3  \\   4 & 0  \\ \end{matrix} \right]}\] and \[\mathbf{3X+2Y=\left[ \begin{matrix}   2 & -2  \\   -1 & 5  \\ \end{matrix} \right]}\]

Ans: Given:

\[2X+3Y=\left[ \begin{matrix}   2 & 3  \\   4 & 0  \\ \end{matrix} \right]\]                               …(1)

\[3X+2Y=\left[ \begin{matrix}   2 & -2  \\   -1 & 5  \\ \end{matrix} \right]\]                            …(2)

Multiplying equation (1) with two we get,

\[2\left( 2X+3Y \right)=2\left[ \begin{matrix}   2 & 3  \\   4 & 0  \\ \end{matrix} \right]\]

\[\Rightarrow \left( 4X+6Y \right)=\left[ \begin{matrix}   4 & 6  \\   8 & 0  \\ \end{matrix} \right]\]                      …(3)

Multiplying equation (2) with three we get,

\[3\left( 3X+2Y \right)=3\left[ \begin{matrix}   2 & -2  \\   -1 & 5  \\ \end{matrix} \right]\]

\[\Rightarrow \left( 9X+6Y \right)=\left[ \begin{matrix}   6 & -6  \\   -3 & 15  \\ \end{matrix} \right]\]                 …(4)

From equation (3) and (4),

\[\left( 4X+6Y \right)-\left( 9X+6Y \right)=\left[ \begin{matrix}   4 & 6  \\   8 & 0  \\ \end{matrix} \right]-\left[ \begin{matrix} 6 & -6  \\   -3 & 15  \\ \end{matrix} \right]\]

\[\Rightarrow -5X=\left[ \begin{matrix}   -2 & 12  \\   11 & -15  \\ \end{matrix} \right]\]

\[\Rightarrow X=\dfrac{-1}{5}\left[ \begin{matrix}   -2 & 12  \\   11 & -15  \\ \end{matrix} \right]\]

\[\therefore X=\left[ \begin{matrix}  \dfrac{2}{5} & \dfrac{-12}{5}  \\   \dfrac{-11}{5} & 3  \\ \end{matrix} \right]\]

Now, since \[2X+3Y=\left[ \begin{matrix}   2 & 3  \\   4 & 0  \\ \end{matrix} \right]\]

\[\therefore 2\left[ \begin{matrix}   \dfrac{2}{5} & \dfrac{-12}{5}  \\   \dfrac{-11}{5} & 3  \\ \end{matrix} \right]+3Y=\left[ \begin{matrix}   2 & 3  \\   4 & 0  \\ \end{matrix} \right]\]

\[\Rightarrow \left[ \begin{matrix} \dfrac{4}{5} & \dfrac{-24}{5}  \\  \dfrac{-22}{5} & 6  \\ \end{matrix} \right]+3Y=\left[ \begin{matrix}   2 & 3  \\   4 & 0  \\ \end{matrix} \right]\]

\[\Rightarrow 3Y=\left[ \begin{matrix}   2 & 3  \\   4 & 0  \\ \end{matrix} \right]-\left[ \begin{matrix}   \dfrac{4}{5} & \dfrac{-24}{5}  \\   \dfrac{-22}{5} & 6  \\ \end{matrix} \right]\]

\[\Rightarrow 3Y=\left[ \begin{matrix}   \dfrac{6}{5} & \dfrac{39}{5}  \\    \dfrac{42}{5} & -6  \\ \end{matrix} \right]\]

\[\therefore Y=\left[ \begin{matrix}  \dfrac{2}{5} & \dfrac{13}{5}  \\   \dfrac{14}{5} & -2  \\ \end{matrix} \right]\]

\[\therefore X=\left[ \begin{matrix}   \dfrac{2}{5} & \dfrac{-12}{5}  \\   \dfrac{-11}{5} & 3  \\ \end{matrix} \right],Y=\left[ \begin{matrix}   \dfrac{2}{5} & \dfrac{13}{5}  \\   \dfrac{14}{5} & -2  \\ \end{matrix} \right]\]


8. Find \[\mathbf{X}\] ,if \[\mathbf{Y=\left[ \begin{matrix}   3 & 2  \\   1 & 4  \\ \end{matrix} \right]}\] and \[\mathbf{2X+Y=\left[ \begin{matrix}   1 & 0  \\   -3 & 2  \\ \end{matrix} \right]}\] .

Ans: Given \[Y=\left[ \begin{matrix}   3 & 2  \\   1 & 4  \\ \end{matrix} \right]\]

And \[2X+Y=\left[ \begin{matrix}   1 & 0  \\   -3 & 2  \\ \end{matrix} \right]\]

\[\Rightarrow 2X+\left[ \begin{matrix}   3 & 2  \\   1 & 4  \\ \end{matrix} \right]=\left[ \begin{matrix}   1 & 0  \\   -3 & 2  \\ \end{matrix} \right]\]

\[\Rightarrow 2X=\left[ \begin{matrix}   1 & 0  \\   -3 & 2  \\ \end{matrix} \right]-\left[ \begin{matrix}   3 & 2  \\   1 & 4  \\ \end{matrix} \right]\]

\[\Rightarrow 2X=\left[ \begin{matrix}   -2 & -2  \\   -4 & -2  \\ \end{matrix} \right]\]

\[\Rightarrow X=\dfrac{1}{2}\left[ \begin{matrix}   -2 & -2  \\   -4 & -2  \\ \end{matrix} \right]\]

\[\therefore X=\left[ \begin{matrix}   -1 & -1  \\   -2 & -1  \\ \end{matrix} \right]\]


9. Find \[X\] and \[Y\] ,if \[\mathbf{2\left[ \begin{matrix}   1 & 3  \\   0 & x  \\ \end{matrix} \right]+\left[ \begin{matrix}   y & 0  \\   1 & 2  \\ \end{matrix} \right]=\left[ \begin{matrix}   5 & 6  \\   1 & 8  \\ \end{matrix} \right]}\]

Ans: Given: \[2\left[ \begin{matrix}   1 & 3  \\   0 & x  \\ \end{matrix} \right]+\left[ \begin{matrix}   y & 0  \\    1 & 2  \\ \end{matrix} \right]=\left[ \begin{matrix}   5 & 6  \\   1 & 8  \\ \end{matrix} \right]\]

\[\Rightarrow \left[ \begin{matrix}   2 & 6  \\   0 & 2x  \\ \end{matrix} \right]+\left[ \begin{matrix}   y & 0  \\   1 & 2  \\ \end{matrix} \right]=\left[ \begin{matrix}   5 & 6  \\   1 & 8  \\ \end{matrix} \right]\]

\[\Rightarrow \left[ \begin{matrix}   2+y & 6+0  \\   0+1 & 2x+2  \\ \end{matrix} \right]=\left[ \begin{matrix}   5 & 6  \\   1 & 8  \\ \end{matrix} \right]\]

\[\Rightarrow \left[ \begin{matrix}   2+y & 6  \\   1 & 2x+2  \\ \end{matrix} \right]=\left[ \begin{matrix}   5 & 6  \\   1 & 8  \\ \end{matrix} \right]\]

Comparing the corresponding elements of these two matrices, we get:

\[2+y=5\]

\[\Rightarrow y=3\]

\[2x+2=8\]

\[\Rightarrow x=3\]

\[\therefore x=3,y=3\]


10. Solve the equation for \[X,Y,Z\] and \[t\] if \[\mathbf{2\left[ \begin{matrix}   x & z  \\   y & t  \\ \end{matrix} \right]+3\left[ \begin{matrix}   1 & -1  \\   0 & 2  \\ \end{matrix} \right]=3\left[ \begin{matrix}   3 & 5  \\   4 & 6  \\ \end{matrix} \right]}\]

Ans: Given: \[2\left[ \begin{matrix}   x & z  \\   y & t  \\ \end{matrix} \right]+3\left[ \begin{matrix}   1 & -1  \\   0 & 2  \\ \end{matrix} \right]=3\left[ \begin{matrix}   3 & 5  \\   4 & 6  \\ \end{matrix} \right]\]

\[\Rightarrow \left[ \begin{matrix}   2x & 2z  \\   2y & 2t  \\ \end{matrix} \right]+\left[ \begin{matrix}   3 & -3  \\   0 & 6  \\ \end{matrix} \right]=\left[ \begin{matrix}   9 & 15  \\   12 & 18  \\ \end{matrix} \right]\]

\[\Rightarrow \left[ \begin{matrix}   2x+3 & 2z-3  \\   2y & 2t+6  \\ \end{matrix} \right]=\left[ \begin{matrix}   9 & 15  \\   12 & 18  \\ \end{matrix} \right]\]

Equating the corresponding elements of these two matrices, we get:

\[2x+3=9\]

\[\Rightarrow x=3\]

\[2y=12\]

\[\Rightarrow y=6\]

\[2z-3=15\]

\[\Rightarrow z=9\]

\[2t+6=18\]

\[\Rightarrow t=6\]

\[\therefore x=3,y=6,z=9,t=6\]


11. If \[\mathbf{x\left[ \begin{matrix}   2  \\   3  \\ \end{matrix} \right]+y\left[ \begin{matrix}   -1  \\   1  \\ \end{matrix} \right]=\left[ \begin{matrix}  10  \\   5  \\ \end{matrix} \right]}\] ,find values of \[x\] and \[y\] .

Ans: Given: \[x\left[ \begin{matrix}   2  \\   3  \\ \end{matrix} \right]+y\left[ \begin{matrix}   -1  \\   1  \\ \end{matrix} \right]=\left[ \begin{matrix}  10  \\   5  \\ \end{matrix} \right]\]

\[\Rightarrow \left[ \begin{matrix}   2x  \\   3x  \\ \end{matrix} \right]+\left[ \begin{matrix}   -y  \\   y  \\ \end{matrix} \right]=\left[ \begin{matrix}   10  \\   5  \\ \end{matrix} \right]\]

\[\Rightarrow \left[ \begin{matrix}   2x-y  \\   3x+y  \\ \end{matrix} \right]=\left[ \begin{matrix}   10  \\   5  \\ \end{matrix} \right]\]

Equating the corresponding elements of these two matrices, we get:

\[2x-y=10\] and

\[3x+y=5\]

Adding these two equations, we get:

\[5x=15\]

\[\Rightarrow x=3\]

Now, since \[3x+y=5\]

\[\Rightarrow y=5-3x\]

\[\Rightarrow y=5-9\]

\[\Rightarrow y=-4\]

\[\therefore x=3,y=-4\]


12. Given \[\mathbf{3\left[ \begin{matrix}   x & y  \\   z & w  \\ \end{matrix} \right]=\left[ \begin{matrix}   x & 6  \\   -1 & 2w  \\ \end{matrix} \right]+\left[ \begin{matrix}   4 & x+y  \\   z+w & 2w+3  \\ \end{matrix} \right]}\] ,find the values of \[x,y,z\] and \[w\] .

Ans: Given: \[3\left[ \begin{matrix}   x & y  \\   z & w  \\ \end{matrix} \right]=\left[ \begin{matrix}   x & 6  \\   -1 & 2w  \\ \end{matrix} \right]+\left[ \begin{matrix}   4 & x+y  \\   z+w & 2w+3  \\ \end{matrix} \right]\] 

\[\Rightarrow \left[ \begin{matrix}   3x & 3y  \\   3z & 3w  \\ \end{matrix} \right]=\left[ \begin{matrix}   x+4 & 6+x+y  \\   -1+z+w & 2w+3  \\ \end{matrix} \right]\]

Equating the corresponding elements of these two matrices, we get:

\[3x=x+4\]

\[\Rightarrow x=2\]

\[3y=6+x+y\]

\[\Rightarrow y=4\]

\[3w=2w+3\]

\[\Rightarrow w=3\]

\[3z=-1+z+w\]

\[\Rightarrow z=1\]

\[\therefore x=2,y=4,z=1,w=3\]


13. If \[\mathbf{F\left( x \right)=\left[ \begin{matrix}   \cos x & -\sin x & 0  \\  \sin x & \cos x & 0  \\   0 & 0 & 1  \\ \end{matrix} \right]}\] , show that \[\mathbf{F\left( x \right)F\left( y \right)=F\left( x+y \right)}\] .

Ans: Here, \[F\left( x \right)=\left[ \begin{matrix}   \cos x & -\sin x & 0  \\  \sin x & \cos x & 0  \\   0 & 0 & 1  \\ \end{matrix} \right]\]

\[F\left( y \right)=\left[ \begin{matrix}   \cos y & -\sin y & 0  \\   \sin y & \cos y & 0 \\   0 & 0 & 1  \\ \end{matrix} \right]\]

\[\therefore F\left( x \right)F\left( y \right)=\left[ \begin{matrix}   \cos x & -\sin x & 0  \\   \sin x & \cos x & 0  \\   0 & 0 & 1  \\ \end{matrix} \right]\left[ \begin{matrix}   \cos y & -\sin y & 0  \\   \sin y & \cos y & 0  \\   0 & 0 & 1  \\ \end{matrix} \right]\]

\[\Rightarrow F\left( x \right)F\left( y \right)=\left[ \begin{matrix}   \cos x\cos y-\sin x\sin y+0 & -\cos x\sin y-\sin x\cos y+0 & 0  \\   \sin x\cos y-\cos x\sin y+0 & -\sin x\sin y+\cos x\cos y+0 & 0  \\   0 & 0 & 1  \\ \end{matrix} \right]\]

\[\Rightarrow F\left( x \right)F\left( y \right)=\left[ \begin{matrix}   \cos \left( x+y \right) & -\sin \left( x+y \right) & 0  \\   \sin \left( x+y \right) & \cos \left( x+y \right) & 0  \\   0 & 0 & 1  \\ \end{matrix} \right]\]                            …(1)

And \[F\left( x+y \right)=\left[ \begin{matrix}   \cos \left( x+y \right) & -\sin \left( x+y \right) & 0  \\   \sin \left( x+y \right) & \cos \left( x+y \right) & 0  \\   0 & 0 & 1  \\ \end{matrix} \right]\]                               …(2)

From equation (1) and (2),

\[F\left( x \right)F\left( y \right)=F\left( x+y \right)\]

Hence proved.


14. Show that

i. \[\mathbf{\left[ \begin{matrix}   5 & -1  \\   6 & 7  \\ \end{matrix} \right]\left[ \begin{matrix}   2 & 1  \\   3 & 4  \\ \end{matrix} \right]\ne \left[ \begin{matrix}   2 & 1  \\   3 & 4  \\ \end{matrix} \right]\left[ \begin{matrix}   5 & -1  \\   6 & 7  \\ \end{matrix} \right]}\]

Ans: LHS: \[\left[ \begin{matrix}   5 & -1  \\   6 & 7  \\ \end{matrix} \right]\left[ \begin{matrix}   2 & 1  \\   3 & 4  \\ \end{matrix} \right]=\left[ \begin{matrix}   5\left( 2 \right)-1\left( 3 \right) & 5\left( 1 \right)-1\left( 4 \right)  \\   6\left( 2 \right)+7\left( 3 \right) & 6\left( 1 \right)+7\left( 4 \right)  \\ \end{matrix} \right]\]

\[\Rightarrow \left[ \begin{matrix}   5 & -1  \\   6 & 7  \\ \end{matrix} \right]\left[ \begin{matrix}   2 & 1  \\   3 & 4  \\ \end{matrix} \right]=\left[ \begin{matrix}   7 & 1  \\   33 & 34  \\ \end{matrix} \right]\]                …(1)

RHS: \[\left[ \begin{matrix}   2 & 1  \\   3 & 4  \\ \end{matrix} \right]\left[ \begin{matrix}   5 & -1  \\   6 & 7  \\ \end{matrix} \right]=\left[ \begin{matrix}   2\left( 5 \right)+1\left( 6 \right) & 2\left( -1 \right)+1\left( 7 \right)  \\   3\left( 5 \right)+4\left( 6 \right) & 3\left( -1 \right)+4\left( 7 \right)  \\ \end{matrix} \right]\]

\[\Rightarrow \left[ \begin{matrix}   2 & 1  \\   3 & 4  \\ \end{matrix} \right]\left[ \begin{matrix}   5 & -1  \\   6 & 7  \\ \end{matrix} \right]=\left[ \begin{matrix}   16 & 5  \\   39 & 25  \\ \end{matrix} \right]\]        …(2)

From equation (1) and (2),

\[LHS\ne RHS\]

Hence proved.

ii. \[\mathbf{\left[ \begin{matrix}   1 & 2 & 3  \\   0 & 1 & 0  \\   1 & 1 & 0  \\ \end{matrix} \right]\left[ \begin{matrix}   -1 & 1 & 0  \\   0 & -1 & 1  \\   2 & 3 & 4  \\ \end{matrix} \right]\ne \left[ \begin{matrix}   -1 & 1 & 0  \\   0 & -1 & 1  \\   2 & 3 & 4  \\ \end{matrix} \right]\left[ \begin{matrix}   1 & 2 & 3  \\   0 & 1 & 0  \\   1 & 1 & 0  \\ \end{matrix} \right]}\]

Ans: LHS: \[\left[ \begin{matrix}   1 & 2 & 3  \\   0 & 1 & 0  \\   1 & 1 & 0  \\ \end{matrix} \right]\left[ \begin{matrix}   -1 & 1 & 0  \\   0 & -1 & 1  \\   2 & 3 & 4  \\ \end{matrix} \right]=\left[ \begin{matrix}   1\left( -1 \right)+2\left( 0 \right)+3\left( 2 \right) & 1\left( 1 \right)+2\left( -1 \right)+3\left( 3 \right) & 1\left( 0 \right)+2\left( 1 \right)+3\left( 4 \right)  \\   0\left( -1 \right)+1\left( 0 \right)+0\left( 2 \right) & 0\left( 1 \right)+1\left( -1 \right)+0\left( 3 \right) & 0\left( 0 \right)+1\left( 1 \right)+0\left( 4 \right)  \\   1\left( -1 \right)+1\left( 0 \right)+0\left( 2 \right) & 1\left( 1 \right)+1\left( -1 \right)+0\left( 3 \right) & 1\left( 0 \right)+1\left( 1 \right)+0\left( 4 \right)  \\ \end{matrix} \right]\]

\[\Rightarrow \left[ \begin{matrix}   1 & 2 & 3  \\   0 & 1 & 0  \\   1 & 1 & 0  \\ \end{matrix} \right]\left[ \begin{matrix}   -1 & 1 & 0  \\   0 & -1 & 1  \\   2 & 3 & 4  \\ \end{matrix} \right]=\left[ \begin{matrix}   5 & 8 & 14  \\   0 & -1 & 1  \\   2 & 3 & 4  \\ \end{matrix} \right]\]                            …(1)

RHS: \[\left[ \begin{matrix}   -1 & 1 & 0  \\   0 & -1 & 1  \\   2 & 3 & 4  \\ \end{matrix} \right]\left[ \begin{matrix}   1 & 2 & 3  \\   0 & 1 & 0  \\   1 & 1 & 0  \\ \end{matrix} \right]=\left[ \begin{matrix}  -1\left( 1 \right)+1\left( 0 \right)+0\left( 1 \right) & -1\left( 2 \right)+1\left( 1 \right)+0\left( 1 \right) & -1\left( 3 \right)+1\left( 0 \right)+0\left( 0 \right)  \\   0\left( 1 \right)-1\left( 0 \right)+1\left( 1 \right) & 0\left( 2 \right)-1\left( 1 \right)+1\left( 1 \right) & 0\left( 3 \right)-1\left( 0 \right)+1\left( 0 \right)  \\  2\left( 1 \right)+3\left( 0 \right)+4\left( 1 \right) & 2\left( 2 \right)+3\left( 1 \right)+4\left( 1 \right) & 2\left( 3 \right)+3\left( 0 \right)+4\left( 0 \right)  \\ \end{matrix} \right]\]

\[\Rightarrow \left[ \begin{matrix}   -1 & 1 & 0  \\   0 & -1 & 1  \\   2 & 3 & 4  \\ \end{matrix} \right]\left[ \begin{matrix}   1 & 2 & 3  \\   0 & 1 & 0  \\   1 & 1 & 0  \\ \end{matrix} \right]=\left[ \begin{matrix}   -1 & -1 & 3  \\   1 & 0 & 0  \\   6 & 11 & 6  \\ \end{matrix} \right]\]                          …(2)

From equation (1) and (2),

\[LHS\ne RHS\]

Hence proved.


15. Find \[\mathbf{{{A}^{2}}-5A+6I}\] if \[\mathbf{A=\left[ \begin{matrix}   2 & 0 & 1  \\   2 & 1 & 3  \\   1 & -1 & 0  \\ \end{matrix} \right]}\] .

Ans: Given: \[A=\left[ \begin{matrix}   2 & 0 & 1  \\   2 & 1 & 3  \\   1 & -1 & 0  \\ \end{matrix} \right]\]

\[{{A}^{2}}=AA\]

\[\therefore {{A}^{2}}=\left[ \begin{matrix}   2 & 0 & 1  \\   2 & 1 & 3  \\   1 & -1 & 0  \\ \end{matrix} \right]\left[ \begin{matrix}   2 & 0 & 1  \\   2 & 1 & 3  \\   1 & -1 & 0  \\ \end{matrix} \right]\]

\[\Rightarrow {{A}^{2}}=\left[ \begin{matrix}   2\left( 2 \right)+0\left( 2 \right)+1\left( 1 \right) & 2\left( 0 \right)+0\left( 1 \right)+1\left( -1 \right) & 2\left( 1 \right)+0\left( 3 \right)+1\left( 0 \right)  \\   2\left( 2 \right)+1\left( 2 \right)+3\left( 1 \right) & 2\left( 0 \right)+1\left( 1 \right)+3\left( -1 \right) & 2\left( 1 \right)+1\left( 3 \right)+3\left( 0 \right)  \\   1\left( 2 \right)-1\left( 2 \right)+0\left( 1 \right) & 1\left( 0 \right)-1\left( 1 \right)+0\left( -1 \right) & 1\left( 1 \right)-1\left( 3 \right)+0\left( 0 \right)  \\ \end{matrix} \right]\]

\[\Rightarrow {{A}^{2}}=\left[ \begin{matrix}   5 & -1 & 2  \\   9 & -2 & 5  \\   0 & -1 & -2  \\ \end{matrix} \right]\]

\[\therefore {{A}^{2}}-5A+6I=\left[ \begin{matrix}   5 & -1 & 2  \\   9 & -2 & 5  \\   0 & -1 & -2  \\ \end{matrix} \right]-5\left[ \begin{matrix}   2 & 0 & 1  \\   2 & 1 & 3  \\   1 & -1 & 0  \\ \end{matrix} \right]+6\left[ \begin{matrix}   1 & 0 & 0  \\   0 & 1 & 0  \\   0 & 0 & 1  \\ \end{matrix} \right]\]

\[\Rightarrow {{A}^{2}}-5A+6I=\left[ \begin{matrix}   5 & -1 & 2  \\   9 & -2 & 5  \\   0 & -1 & -2  \\ \end{matrix} \right]-\left[ \begin{matrix}   10 & 0 & 5  \\   10 & 5 & 15  \\   5 & -5 & 0  \\ \end{matrix} \right]+\left[ \begin{matrix}   6 & 0 & 0  \\   0 & 6 & 0  \\   0 & 0 & 6  \\ \end{matrix} \right]\]

\[\Rightarrow {{A}^{2}}-5A+6I=\left[ \begin{matrix}   5-10 & -1-0 & 2-5  \\   9-10 & -2-5 & 5-15  \\   0-5 & -1+5 & -2-0  \\ \end{matrix} \right]+\left[ \begin{matrix}   6 & 0 & 0  \\   0 & 6 & 0  \\   0 & 0 & 6  \\ \end{matrix} \right]\]

\[\Rightarrow {{A}^{2}}-5A+6I=\left[ \begin{matrix}   -5 & -1 & -3  \\   -1 & -7 & -10 \\   -5 & 4 & -2  \\ \end{matrix} \right]+\left[ \begin{matrix}   6 & 0 & 0  \\   0 & 6 & 0  \\   0 & 0 & 6  \\ \end{matrix} \right]\]

\[\Rightarrow {{A}^{2}}-5A+6I=\left[ \begin{matrix}   1 & -1 & -3  \\   -1 & -1 & -10 \\   -5 & 4 & 4  \\ \end{matrix} \right]\]


16. If \[\mathbf{A=\left[ \begin{matrix}   1 & 0 & 2  \\   0 & 2 & 1  \\   2 & 0 & 3  \\ \end{matrix} \right]}\] , prove that \[\mathbf{{{A}^{3}}-6{{A}^{2}}+7A+21=0}\] .

Ans: Given: \[A=\left[ \begin{matrix}   1 & 0 & 2  \\   0 & 2 & 1  \\   2 & 0 & 3  \\ \end{matrix} \right]\]

\[{{A}^{2}}=AA\]

\[\therefore {{A}^{2}}=\left[ \begin{matrix}   1 & 0 & 2  \\   0 & 2 & 1  \\   2 & 0 & 3  \\ \end{matrix} \right]\left[ \begin{matrix}   1 & 0 & 2  \\   0 & 2 & 1  \\   2 & 0 & 3  \\ \end{matrix} \right]\]

\[\Rightarrow {{A}^{2}}=\left[ \begin{matrix}   1+0+4 & 0+0+0 & 2+0+6  \\   0+0+2 & 0+4+0 & 0+2+3  \\   2+0+6 & 0+0+0 & 4+0+9  \\ \end{matrix} \right]\]

\[\Rightarrow {{A}^{2}}=\left[ \begin{matrix}   5 & 0 & 8  \\   2 & 4 & 5  \\   8 & 0 & 13  \\ \end{matrix} \right]\]

Now, \[{{A}^{3}}={{A}^{2}}\cdot A\]

\[\therefore {{A}^{3}}=\left[ \begin{matrix}   5 & 0 & 8  \\   2 & 4 & 5  \\   8 & 0 & 13  \\ \end{matrix} \right]\left[ \begin{matrix}   1 & 0 & 2  \\   0 & 2 & 1  \\   2 & 0 & 3  \\ \end{matrix} \right]\]

\[\Rightarrow {{A}^{3}}=\left[ \begin{matrix}   5+0+16 & 0+0+0 & 10+0+24  \\   2+0+10 & 0+8+0 & 4+4+15  \\   8+0+26 & 0+0+0 & 16+0+39  \\ \end{matrix} \right]\]

\[\Rightarrow {{A}^{3}}=\left[ \begin{matrix}   21 & 0 & 34  \\   12 & 8 & 23  \\   34 & 0 & 55  \\ \end{matrix} \right]\]

\[\therefore {{A}^{3}}-6{{A}^{2}}+7A+21=\left[ \begin{matrix}   21 & 0 & 34  \\   12 & 8 & 23  \\   34 & 0 & 55  \\ \end{matrix} \right]-6\left[ \begin{matrix}   5 & 0 & 8  \\   2 & 4 & 5  \\   8 & 0 & 13  \\ \end{matrix} \right]+7\left[ \begin{matrix}   1 & 0 & 2  \\   0 & 2 & 1  \\   2 & 0 & 3  \\ \end{matrix} \right]+2\left[ \begin{matrix}   1 & 0 & 0  \\   0 & 1 & 0  \\   0 & 0 & 1  \\ \end{matrix} \right]\]

\[\Rightarrow {{A}^{3}}-6{{A}^{2}}+7A+21=\left[ \begin{matrix}   21 & 0 & 34  \\   12 & 8 & 23  \\   34 & 0 & 55  \\ \end{matrix} \right]-\left[ \begin{matrix}   30 & 0 & 48  \\   12 & 24 & 30  \\   48 & 0 & 78  \\ \end{matrix} \right]+\left[ \begin{matrix}   7 & 0 & 14  \\   0 & 14 & 7  \\   14 & 0 & 21  \\ \end{matrix} \right]+\left[ \begin{matrix}   2 & 0 & 0  \\   0 & 2 & 0  \\   0 & 0 & 2  \\ \end{matrix} \right]\]

\[\Rightarrow {{A}^{3}}-6{{A}^{2}}+7A+21=\left[ \begin{matrix}   21-30+7+2 & 0-0+0+0 & 34-48+14+0  \\   12-12+0+0 & 8-24+14+2 & 23-30+7+0  \\   34-48+14+0 & 0-0+0+0 & 55-78+21+2  \\ \end{matrix} \right]\]

\[\Rightarrow {{A}^{3}}-6{{A}^{2}}+7A+21=\left[ \begin{matrix}   0 & 0 & 0  \\   0 & 0 & 0  \\   0 & 0 & 0  \\ \end{matrix} \right]\]

\[\therefore {{A}^{3}}-6{{A}^{2}}+7A+21=0\]

Hence proved.


17.  If \[\mathbf{A=\left[ \begin{matrix}   3 & -2  \\   4 & -2  \\ \end{matrix} \right]}\] and \[\mathbf{I=\left[ \begin{matrix}   1 & 0  \\   0 & 1  \\ \end{matrix} \right]}\] , find \[\mathbf{k}\] so that \[\mathbf{{{A}^{2}}=kA-21}\] .

Ans: Given:\[A=\left[ \begin{matrix}   3 & -2  \\   4 & -2  \\ \end{matrix} \right]\]

\[{{A}^{2}}=AA\]

\[\Rightarrow {{A}^{2}}=\left[ \begin{matrix}   3 & -2  \\   4 & -2  \\ \end{matrix} \right]\left[ \begin{matrix}   3 & -2  \\   4 & -2  \\ \end{matrix} \right]\]

\[\Rightarrow {{A}^{2}}=\left[ \begin{matrix}   9-8 & -6+4  \\   12-8 & -8+4  \\ \end{matrix} \right]\]

\[\Rightarrow {{A}^{2}}=\left[ \begin{matrix}   1 & -2  \\   4 & -4  \\ \end{matrix} \right]\]

Now, \[{{A}^{2}}=kA-21\]

\[\Rightarrow \left[ \begin{matrix}   1 & -2  \\   4 & -4  \\ \end{matrix} \right]=k\left[ \begin{matrix}   3 & -2  \\   4 & -2  \\ \end{matrix} \right]-2\left[ \begin{matrix}   1 & 0  \\   0 & 1  \\ \end{matrix} \right]\]

\[\Rightarrow \left[ \begin{matrix}   1 & -2  \\   4 & -4  \\ \end{matrix} \right]=\left[ \begin{matrix}   3k & -2k  \\   4k & -2k  \\ \end{matrix} \right]-\left[ \begin{matrix}   2 & 0  \\   0 & 2  \\ \end{matrix} \right]\]

\[\Rightarrow \left[ \begin{matrix}   1 & -2  \\   4 & -4  \\ \end{matrix} \right]=\left[ \begin{matrix}   3k-2 & -2k  \\   4k & -2k-2  \\ \end{matrix} \right]\]

Equating the corresponding elements, we have:

\[3k-2=1\]

\[\Rightarrow k=1\]

Thus, the value of \[k\] is \[1\].


18. If \[\mathbf{A=\left[ \begin{matrix}   0 & -\tan \dfrac{\alpha }{2}  \\   \tan \dfrac{\alpha }{2} & 0  \\ \end{matrix} \right]}\] and \[I\] is the identity matrix of order \[2\] , show that \[\mathbf{I+A=\left( I-A \right)\left[ \begin{matrix}    \cos \alpha  & -\sin \alpha   \\   \sin \alpha  & \cos \alpha   \\ \end{matrix} \right]}\]

Ans: Given: \[A=\left[ \begin{matrix}   0 & -\tan \dfrac{\alpha }{2}  \\   \tan \dfrac{\alpha }{2} & 0  \\ \end{matrix} \right]\]

LHS: \[I+A=\left[ \begin{matrix}   1 & 0  \\   0 & 1  \\ \end{matrix} \right]+\left[ \begin{matrix}   0 & -\tan \dfrac{\alpha }{2}  \\   \tan \dfrac{\alpha }{2} & 0  \\ \end{matrix} \right]\]

\[\Rightarrow I+A=\left[ \begin{matrix}   1 & -\tan \dfrac{\alpha }{2}  \\   \tan \dfrac{\alpha }{2} & 1  \\ \end{matrix} \right]\]                                           …(1)

RHS: \[\left( I-A \right)\left[ \begin{matrix}   \cos \alpha  & -\sin \alpha   \\   \sin \alpha  & \cos \alpha   \\ \end{matrix} \right]=\left( \left[ \begin{matrix}   1 & 0  \\   0 & 1  \\ \end{matrix} \right]-\left[ \begin{matrix}   0 & -\tan \dfrac{\alpha }{2}  \\   \tan \dfrac{\alpha }{2} & 0  \\ \end{matrix} \right] \right)\left[ \begin{matrix}   \cos \alpha  & -\sin \alpha   \\   \sin \alpha  & \cos \alpha   \\ \end{matrix} \right]\]

\[\Rightarrow \left( I-A \right)\left[ \begin{matrix}   \cos \alpha  & -\sin \alpha   \\   \sin \alpha  & \cos \alpha   \\ \end{matrix} \right]=\left[ \begin{matrix}  1 & \tan \dfrac{\alpha }{2}  \\   -\tan \dfrac{\alpha }{2} & 1  \\ \end{matrix} \right]\left[ \begin{matrix}   \cos \alpha  & -\sin \alpha   \\   \sin \alpha  & \cos \alpha   \\ \end{matrix} \right]\]

\[\Rightarrow \left( I-A \right)\left[ \begin{matrix}   \cos \alpha  & -\sin \alpha   \\    \sin \alpha  & \cos \alpha   \\ \end{matrix} \right]=\left[ \begin{matrix}   \cos \alpha +\sin \alpha \tan \dfrac{\alpha }{2} & -\sin \alpha +\cos \alpha \tan \dfrac{\alpha }{2}  \\   -\cos \alpha \tan \dfrac{\alpha }{2}+\sin \alpha  & \sin \alpha \tan \dfrac{\alpha }{2}+\cos \alpha   \\ \end{matrix} \right]\]

\[\Rightarrow \left( I-A \right)\left[ \begin{matrix}   \cos \alpha  & -\sin \alpha   \\   \sin \alpha  & \cos \alpha   \\ \end{matrix} \right]=\left[ \begin{matrix}   1-2{{\sin }^{2}}\dfrac{\alpha }{2}+2\sin \dfrac{\alpha }{2}-\cos \dfrac{\alpha }{2}\tan \dfrac{\alpha }{2} & -2\sin \dfrac{\alpha }{2}\cos \dfrac{\alpha }{2}+\left( 2{{\cos }^{2}}\dfrac{\alpha }{2}-1 \right)\tan \dfrac{\alpha }{2}  \\   -\left( 2{{\cos }^{2}}\dfrac{\alpha }{2}-1 \right)\tan \dfrac{\alpha }{2}+2\sin \dfrac{\alpha }{2}\cos \dfrac{\alpha }{2} & 2\sin \dfrac{\alpha }{2}\cos \dfrac{\alpha }{2}\tan \dfrac{\alpha }{2}+1-2{{\sin }^{2}}\dfrac{\alpha }{2}  \\ \end{matrix} \right]\]

\[\Rightarrow \left( I-A \right)\left[ \begin{matrix}   \cos \alpha  & -\sin \alpha   \\   \sin \alpha  & \cos \alpha   \\ \end{matrix} \right]=\left[ \begin{matrix}   1-2{{\sin }^{2}}\dfrac{\alpha }{2}+2{{\sin }^{2}}\dfrac{\alpha }{2} & -2\sin \dfrac{\alpha }{2}\cos \dfrac{\alpha }{2}+2\sin \dfrac{\alpha }{2}\cos \dfrac{\alpha }{2}-\tan \dfrac{\alpha }{2}  \\   -\sin \dfrac{\alpha }{2}\cos \dfrac{\alpha }{2}+\tan \dfrac{\alpha }{2}+2\sin \dfrac{\alpha }{2}\cos \dfrac{\alpha }{2} & 2{{\sin }^{2}}\dfrac{\alpha }{2}+1-2{{\sin }^{2}}\dfrac{\alpha }{2}  \\ \end{matrix} \right]\]

\[\Rightarrow \left( I-A \right)\left[ \begin{matrix}   \cos \alpha  & -\sin \alpha   \\    \sin \alpha  & \cos \alpha   \\ \end{matrix} \right]=\left[ \begin{matrix}   1 & -\tan \dfrac{\alpha }{2}  \\    \tan \dfrac{\alpha }{2} & 1  \\ \end{matrix} \right]\]                                        …(2)

Thus, from equation (1) and (2).

\[LHS=RHS\]

Hence proved.


19. A trust fund has Rs \[30,000\] that must be invested in two different types of bonds. The first bond pays \[5%\] interest per year, and the second bond pays \[7%\] interest per year. Using matrix multiplication, determine how to divide Rs \[30,000\] among the two types of bonds. If the trust fund must obtain an annual total interest of:

i. Rs \[\mathbf{1,800}\]

Ans: Let Rs \[x\] be invested in the first round.

Then, the sum of money invested in the second bond pays Rs \[\left( 30000-x \right)\]

It is given that the first bond pays \[5%\] interest per year, and the second bond pays \[7%\] interest per year.

We know that,

Simple interest for one year is \[\dfrac{principle\times rate}{100}\] .

Therefore, in order to obtain an annual total interest of Rs \[1,800\] ,

\[\left[ \begin{matrix}   x & \left( 30000-x \right)  \\ \end{matrix} \right]\left[ \begin{matrix}   \dfrac{5}{100}  \\   \dfrac{7}{100}  \\ \end{matrix} \right]=1800\]

\[\Rightarrow \dfrac{5x}{100}+\dfrac{7\left( 30000-x \right)}{100}=1800\]

\[\Rightarrow 210000-2x=180000\]

\[\Rightarrow x=15000\]

Thus, in order to obtain an annual total interest of Rs \[1,800\] , the trust fund should invest Rs \[15000\] in the first bond and the remaining Rs \[15000\] in the second bond.

ii. Rs \[\mathbf{2,000}\]

Ans: Let Rs \[x\] be invested in the first round.

Then, the sum of money invested in the second bond pays Rs \[\left( 30000-x \right)\]

It is given that the first bond pays \[5%\] interest per year, and the second bond pays \[7%\] interest per year.

We know that,

Simple interest for one year is \[\dfrac{principle\times rate}{100}\] .

Therefore, in order to obtain an annual total interest of Rs \[2,000\] ,

\[\left[ \begin{matrix}   x & \left( 30000-x \right)  \\ \end{matrix} \right]\left[ \begin{matrix}   \dfrac{5}{100}  \\   \dfrac{7}{100}  \\ \end{matrix} \right]=2000\]

\[\Rightarrow \dfrac{5x}{100}+\dfrac{7\left( 30000-x \right)}{100}=2000\]

\[\Rightarrow 210000-2x=200000\]

\[\Rightarrow x=5000\]

Thus, in order to obtain an annual total interest of Rs \[2,000\] , the trust fund should invest Rs \[5000\] in the first bond and the remaining Rs \[25000\] in the second bond.


20. The bookshop of a particular school has \[10\] dozen chemistry books, \[8\] dozen physics books, \[10\] dozen economics books. Their selling prices are Rs \[80\], Rs \[60\] and Rs \[40\] each respectively. Find the total amount the bookshop will receive from selling all the books using matrix algebra.

Ans: The total amount of money that will be received from the sale of all these books can be represented in the form of a matrix as:

\[12\left[ \begin{matrix}    10 & 8 & 10  \\ \end{matrix} \right]\left[ \begin{matrix}   80  \\   60  \\   40  \\ \end{matrix} \right]\]

\[\Rightarrow 12\left[ 10\times 80+8\times 60+10\times 40 \right]\]

\[\Rightarrow 12\left[ 1680 \right]\]

\[\Rightarrow 20160\]

Therefore, the bookshop will receive Rs \[20160\] from the sale.


21. Assume \[X,Y,Z,W\] and \[P\] are the matrices of order \[\mathbf{2\times n,3\times k,2\times p,n\times 3}\] and \[\mathbf{p\times k}\] respectively. The restriction on \[\mathbf{n,k,p}\] so that \[\mathbf{PY+WY}\] will be defined are:

  1. \[\mathbf{k=3,p=n}\]

  2. \[\mathbf{k}\] is arbitrary, \[\mathbf{p=2}\]

  3. \[\mathbf{p}\] is arbitrary, \[\mathbf{k=3}\]

  4. \[\mathbf{k=2,p=3}\]

Ans: Matrices \[P\] and \[Y\] are of the orders \[p\times k\] and \[3\times k\] respectively.

Therefore, the matrix \[PY\] will be defined if \[k=3\] .

Also, \[PY\] will be of order \[p\times k\] .

Since the number of columns in matrix \[W\] is equal to the number of rows in matrix \[Y\] .

Therefore, matrix \[WY\] is well-defined and is of the order \[n\times k\] .

Moreover, Matrices \[PY\] and \[WY\] can be added only when their orders are the same.

But \[PY\] is of the order \[p\times k\] and \[WY\] is of the order \[n\times k\] .

Therefore, we must have \[p=n\] .

\[\therefore p=n,k=3\] are the restrictions on \[n,k,p\] so that \[PY+WY\] will be defined.

Thus, option (A) is correct.


22. Assume \[\mathbf{X,Y,Z,W}\] and \[\mathbf{P}\] are matrices of order \[\mathbf{2\times n,3\times k,2\times p,n\times 3}\] and \[\mathbf{p\times k}\] respectively. If \[\mathbf{p=n}\] , then the order of the matrix \[\mathbf{7X-5Z}\] is:

  1. \[\mathbf{p\times 2}\]

  2. \[\mathbf{2\times n}\]

  3. \[\mathbf{n\times 3}\]

  4. \[\mathbf{p\times n}\]

Ans: Matrix \[X\] is of the order \[2\times n\] .

Thus, matrix \[7X\] is also of the same order.

Since, \[p=n\]

Matrix \[Z\] is of the order \[2\times p\] or \[2\times n\] .

Thus, matrix \[5Z\] is also of the same order.

Since, both the matrices \[7X\] and \[5Z\] are of the same order \[2\times n\].

Therefore, \[7X-5Z\] is well defined and is of the order \[2\times n\] .

Thus, option (B) is correct.


Exercise 3.3

1. Find the transpose of each of the following matrices:

i. \[\left[ \begin{matrix}   5  \\   \dfrac{1}{2}  \\   -1  \\ \end{matrix} \right]\]

Ans: The transpose of a matrix is obtained by changing its rows into columns and its columns into rows.

Thus, if \[A=\left[ \begin{matrix}  5  \\   \dfrac{1}{2}  \\   -1  \\ \end{matrix} \right]\] , then \[{{A}^{T}}=\left[ \begin{matrix}   5 & \dfrac{1}{2} & -1  \\ \end{matrix} \right]\]

ii. \[\left[ \begin{matrix}   1 & -1  \\   2 & 3  \\ \end{matrix} \right]\]

Ans: The transpose of a matrix is obtained by changing its rows into columns and its columns into rows.

Thus, if \[A=\left[ \begin{matrix}  1 & -1  \\   2 & 3  \\ \end{matrix} \right]\] , then \[{{A}^{T}}=\left[ \begin{matrix}   1 & 2  \\   -1 & 3  \\ \end{matrix} \right]\]

iii. \[\left[ \begin{matrix}   -1 & 5 & 6  \\   \sqrt{3} & 5 & 6  \\    2 & 3 & -1  \\ \end{matrix} \right]\]

Ans: The transpose of a matrix is obtained by changing its rows into columns and its columns into rows.

Thus, if \[A=\left[ \begin{matrix}   -1 & 5 & 6  \\   \sqrt{3} & 5 & 6  \\   2 & 3 & -1  \\ \end{matrix} \right]\] , then \[{{A}^{T}}=\left[ \begin{matrix}   -1 & \sqrt{3} & 2  \\   5 & 5 & 3  \\   6 & 6 & -1  \\ \end{matrix} \right]\]


2. If \[A=\left[ \begin{matrix}   -1 & 2 & 3  \\   5 & 7 & 9  \\   -2 & 1 & 1  \\ \end{matrix} \right]\] and \[B=\left[ \begin{matrix}   -4 & 1 & -5  \\   1 & 2 & 0  \\   1 & 3 & 1  \\ \end{matrix} \right]\] , then verify that

i. \[\left( A+B \right)'=A'+B'\]

Ans: Given that: \[A=\left[ \begin{matrix}   -1 & 2 & 3  \\   5 & 7 & 9  \\   -2 & 1 & 1  \\ \end{matrix} \right]\] and \[B=\left[ \begin{matrix}   -4 & 1 & -5  \\   1 & 2 & 0  \\   1 & 3 & 1  \\ \end{matrix} \right]\] 

Thus, we have \[A'=\left[ \begin{matrix}   -1 & 5 & -2  \\   2 & 7 & 1  \\   3 & 9 & 1  \\ \end{matrix} \right]\] and \[B'=\left[ \begin{matrix}   -4 & 1 & 1  \\   1 & 2 & 3  \\   -5 & 0 & 1  \\ \end{matrix} \right]\]

\[\left( A+B \right)=\left[ \begin{matrix}   -1 & 2 & 3  \\   5 & 7 & 9  \\   -2 & 1 & 1  \\ \end{matrix} \right]+\left[ \begin{matrix}   -4 & 1 & -5  \\   1 & 2 & 0  \\   1 & 3 & 1  \\ \end{matrix} \right]\]

\[\therefore \left( A+B \right)=\left[ \begin{matrix}   -5 & 3 & -2  \\   6 & 9 & 9  \\   -1 & 4 & 2  \\ \end{matrix} \right]\]

\[\Rightarrow \left( A+B \right)'=\left[ \begin{matrix}   -5 & 6 & -1  \\   3 & 9 & 4  \\   -2 & 9 & 2  \\ \end{matrix} \right]\]                                   …(1)

And \[A'+B'=\left[ \begin{matrix}   -1 & 5 & -2  \\   2 & 7 & 1  \\   3 & 9 & 1  \\ \end{matrix} \right]+\left[ \begin{matrix}   -4 & 1 & 1  \\   1 & 2 & 3  \\   -5 & 0 & 1  \\ \end{matrix} \right]\]

\[\therefore A'+B'=\left[ \begin{matrix}   -5 & 6 & -1  \\   3 & 9 & 4  \\   -2 & 9 & 2  \\ \end{matrix} \right]\]                                         …(2)

Thus, from equation (1) and (2),

\[\left( A+B \right)'=A'+B'\]

Hence proved.

ii. \[\left( A-B \right)'=A'-B'\]

Ans: Given that: \[A=\left[ \begin{matrix}   -1 & 2 & 3  \\   5 & 7 & 9  \\   -2 & 1 & 1  \\ \end{matrix} \right]\] and \[B=\left[ \begin{matrix}   -4 & 1 & -5  \\   1 & 2 & 0  \\   1 & 3 & 1  \\ \end{matrix} \right]\] 

Thus, we have \[A'=\left[ \begin{matrix}   -1 & 5 & -2  \\   2 & 7 & 1  \\   3 & 9 & 1  \\ \end{matrix} \right]\] and \[B'=\left[ \begin{matrix}   -4 & 1 & 1  \\   1 & 2 & 3  \\   -5 & 0 & 1  \\ \end{matrix} \right]\]

\[\left( A-B \right)=\left[ \begin{matrix}   -1 & 2 & 3  \\   5 & 7 & 9  \\   -2 & 1 & 1  \\ \end{matrix} \right]-\left[ \begin{matrix}  -4 & 1 & -5  \\   1 & 2 & 0  \\   1 & 3 & 1  \\ \end{matrix} \right]\]

\[\therefore \left( A-B \right)=\left[ \begin{matrix}   3 & 1 & 8  \\   4 & 5 & 9  \\   -3 & -2 & 0  \\ \end{matrix} \right]\]

\[\Rightarrow \left( A-B \right)'=\left[ \begin{matrix}   3 & 4 & -3  \\   1 & 5 & -2 \\   8 & 9 & 0  \\ \end{matrix} \right]\]                                   …(1)

And \[A'-B'=\left[ \begin{matrix}   -1 & 5 & -2  \\   2 & 7 & 1  \\   3 & 9 & 1  \\ \end{matrix} \right]-\left[ \begin{matrix}   -4 & 1 & 1  \\   1 & 2 & 3  \\   -5 & 0 & 1  \\ \end{matrix} \right]\]

\[\therefore A'-B'=\left[ \begin{matrix}   3 & 4 & -3  \\   1 & 5 & -2  \\   8 & 9 & 0  \\ \end{matrix} \right]\]                                         …(2)

Thus, from equation (1) and (2),

\[\left( A-B \right)'=A'-B'\]

Hence proved.


3. If \[A'=\left[ \begin{matrix}   3 & 4  \\   -1 & 2  \\   0 & 1  \\ \end{matrix} \right]\] and \[B=\left[ \begin{matrix}   -1 & 2 & 1  \\   1 & 2 & 3  \\ \end{matrix} \right]\] , then verify that

i. \[\left( A+B \right)'=A'+B'\]

Ans: We know that, \[A=\left( A' \right)'\]

Thus, we have: \[A=\left[ \begin{matrix}   3 & -1 & 0  \\   4 & 2 & 1  \\ \end{matrix} \right]\]

And \[B'=\left[ \begin{matrix}   -1 & 1  \\   2 & 2  \\   1 & 3  \\ \end{matrix} \right]\]

\[\therefore A+B=\left[ \begin{matrix}   3 & -1 & 0  \\   4 & 2 & 1  \\ \end{matrix} \right]+\left[ \begin{matrix}   -1 & 2 & 1  \\   1 & 2 & 3  \\ \end{matrix} \right]\]

\[\Rightarrow A+B=\left[ \begin{matrix}   2 & 1 & 1  \\   5 & 4 & 4  \\ \end{matrix} \right]\]

\[\therefore \left( A+B \right)'=\left[ \begin{matrix}   2 & 5  \\   1 & 4  \\   1 & 4  \\ \end{matrix} \right]\]                                …(1)

\[A'+B'=\left[ \begin{matrix}   3 & 4  \\   -1 & 2  \\   0 & 1  \\ \end{matrix} \right]+\left[ \begin{matrix}   -1 & 1  \\   2 & 2  \\   1 & 3  \\ \end{matrix} \right]\]

\[\therefore A'+B'=\left[ \begin{matrix}   2 & 5  \\   1 & 4  \\   1 & 4  \\ \end{matrix} \right]\]                                      …(2)

Thus, from equation (1) and (2),

\[\left( A+B \right)'=A'+B'\]

Hence proved.

ii. \[\left( A-B \right)'=A'-B'\]

Ans: We know that, \[A=\left( A' \right)'\]

Thus, we have: \[A=\left[ \begin{matrix}  3 & -1 & 0  \\   4 & 2 & 1  \\ \end{matrix} \right]\]

And \[B'=\left[ \begin{matrix}   -1 & 1  \\   2 & 2  \\   1 & 3  \\ \end{matrix} \right]\]

\[\therefore A-B=\left[ \begin{matrix}   3 & -1 & 0  \\   4 & 2 & 1  \\ \end{matrix} \right]-\left[ \begin{matrix}   -1 & 2 & 1  \\   1 & 2 & 3  \\ \end{matrix} \right]\]

\[\Rightarrow A-B=\left[ \begin{matrix}   4 & -3 & -1  \\   3 & 0 & -2  \\ \end{matrix} \right]\]

\[\therefore \left( A-B \right)'=\left[ \begin{matrix}   4 & 3  \\   -3 & 0  \\   -1 & -2  \\ \end{matrix} \right]\]                                …(1)

\[A'-B'=\left[ \begin{matrix}   3 & 4  \\   -1 & 2  \\   0 & 1  \\ \end{matrix} \right]-\left[ \begin{matrix}   -1 & 1  \\   2 & 2  \\   1 & 3  \\ \end{matrix} \right]\]

\[\therefore A'-B'=\left[ \begin{matrix}   4 & 3  \\   -3 & 0  \\   -1 & -2  \\ \end{matrix} \right]\]                                      …(2)

Thus, from equation (1) and (2),

\[\left( A-B \right)'=A'-B'\]

Hence proved.


4. If \[A'=\left[ \begin{matrix}   -2 & 3  \\   1 & 2  \\ \end{matrix} \right]\] and \[B=\left[ \begin{matrix}   -1 & 0  \\   1 & 2  \\ \end{matrix} \right]\] , then ( A+2B \right)' .

Ans: We know that, \[A=\left( A' \right)'\]

\[\therefore A=\left[ \begin{matrix}   -2 & 1  \\   3 & 2  \\ \end{matrix} \right]\]

\[\therefore A+2B=\left[ \begin{matrix}   -2 & 1  \\   3 & 2  \\ \end{matrix} \right]+2\left[ \begin{matrix}   -1 & 0  \\   1 & 2  \\ \end{matrix} \right]\]

\[\Rightarrow A+2B=\left[ \begin{matrix}   -2 & 1  \\   3 & 2  \\ \end{matrix} \right]+\left[ \begin{matrix}   -2 & 0  \\   2 & 4  \\ \end{matrix} \right]\]

\[\Rightarrow A+2B=\left[ \begin{matrix}   -4 & 1  \\   5 & 6  \\ \end{matrix} \right]\]

\[\therefore \left( A+2B \right)'=\left[ \begin{matrix}   -4 & 5  \\   1 & 6  \\ \end{matrix} \right]\]


5. For the matrices \[A\] and \[B\] , verify that \[\left( AB \right)'=B'A'\] where

i. \[A=\left[ \begin{matrix}   1  \\   -4  \\   3  \\ \end{matrix} \right],B=\left[ \begin{matrix}    -1 & 2 & 1  \\ \end{matrix} \right]\]

Ans: LHS: \[AB=\left[ \begin{matrix}   1  \\   -4  \\   3  \\ \end{matrix} \right]\left[ \begin{matrix}   -1 & 2 & 1  \\ \end{matrix} \right]\]

\[\Rightarrow AB=\left[ \begin{matrix}   -1 & 2 & 1  \\   4 & -8 & -4  \\   -3 & 6 & 3  \\ \end{matrix} \right]\]

\[\therefore \left( AB \right)'=\left[ \begin{matrix}   -1 & 4 & -3  \\   2 & -8 & 6 \\   1 & -4 & 3  \\ \end{matrix} \right]\]                                 …(1)

Now, \[A'=\left[ \begin{matrix}   1 & -4 & 3  \\ \end{matrix} \right]\]

And \[B'=\left[ \begin{matrix}   -1  \\   2  \\   1  \\ \end{matrix} \right]\]

RHS: \[\therefore B'A'=\left[ \begin{matrix}   -1  \\   2  \\   1  \\ \end{matrix} \right]\left[ \begin{matrix}   1 & -4 & 3  \\ \end{matrix} \right]\]                      

 \[\therefore B'A'=\left[ \begin{matrix}   -1 & 4 & -3  \\   2 & -8 & 6  \\   1 & -4 & 3  \\ \end{matrix} \right]\]                                     …(2)

Thus, from equation (1) and (2),

\[\left( AB \right)'=B'A'\]

Hence proved.

ii. \[A=\left[ \begin{matrix}   0  \\   1  \\ 2  \\ \end{matrix} \right], B=\left[ \begin{matrix}   1 & 5 & 7  \\ \end{matrix} \right]\]

Ans: LHS: \[AB=\left[ \begin{matrix}   0  \\   1  \\   2  \\ \end{matrix} \right]\left[ \begin{matrix}   1 & 5 & 7  \\ \end{matrix} \right]\]

\[\Rightarrow AB=\left[ \begin{matrix}   0 & 1 & 2  \\   0 & 5 & 10  \\   0 & 7 & 14  \\ \end{matrix} \right]\]

\[\therefore \left( AB \right)'=\left[ \begin{matrix}   0 & 1 & 2  \\   0 & 5 & 10  \\   0 & 7 & 14  \\ \end{matrix} \right]\]                                 …(1)

Now, \[A'=\left[ \begin{matrix}   0 & 1 & 2  \\ \end{matrix} \right]\]

And \[B'=\left[ \begin{matrix}   1  \\   5  \\   7  \\ \end{matrix} \right]\]

RHS: \[\therefore B'A'=\left[ \begin{matrix}   1  \\   5  \\   7  \\ \end{matrix} \right]\left[ \begin{matrix}   0 & 1 & 2  \\ \end{matrix} \right]\]

\[\therefore B'A'=\left[ \begin{matrix}   0 & 1 & 2  \\   0 & 5 & 10  \\   0 & 7 & 14  \\ \end{matrix} \right]\]                                  …(2)

Thus, from equation (1) and (2),

\[\left( AB \right)'=B'A'\]

Hence proved.


6. If 

i. \[A=\left[ \begin{matrix}   \cos \alpha  & \sin \alpha   \\   -\sin \alpha  & \cos \alpha   \\ \end{matrix} \right]\] , then verify that \[A'A=I\]

Ans: Here, \[A'=\left[ \begin{matrix}   \cos \alpha  & -\sin \alpha   \\   \sin \alpha  & \cos \alpha   \\ \end{matrix} \right]\]

\[\therefore A'A=\left[ \begin{matrix}   \cos \alpha  & -\sin \alpha   \\   \sin \alpha  & \cos \alpha   \\ \end{matrix} \right]\left[ \begin{matrix}   \cos \alpha  & \sin \alpha   \\   -\sin \alpha  & \cos \alpha   \\ \end{matrix} \right]\]

\[\Rightarrow A'A=\left[ \begin{matrix}   {{\cos }^{2}}\alpha +{{\sin }^{2}}\alpha  & \sin \alpha \cos \alpha -\sin \alpha \cos \alpha   \\   \sin \alpha \cos \alpha -\sin \alpha \cos \alpha  & {{\cos }^{2}}\alpha +{{\sin }^{2}}\alpha   \\ \end{matrix} \right]\]

\[\therefore A'A=\left[ \begin{matrix}   1 & 0  \\   0 & 1  \\ \end{matrix} \right]\]

\[\therefore A'A=I\]

Hence proved.

ii. \[A=\left[ \begin{matrix}   \sin \alpha  & \cos \alpha   \\   -\cos \alpha  & \sin \alpha   \\ \end{matrix} \right]\] , then verify that \[A'A=I\]

Ans: Here, \[A'=\left[ \begin{matrix}   \sin \alpha  & -\cos \alpha   \\   \cos \alpha  & \sin \alpha   \\ \end{matrix} \right]\]

\[\therefore A'A=\left[ \begin{matrix}   \sin \alpha  & -\cos \alpha   \\    \cos \alpha  & \sin \alpha   \\ \end{matrix} \right]\left[ \begin{matrix}   \sin \alpha  & \cos \alpha   \\   -\cos \alpha  & \sin \alpha   \\ \end{matrix} \right]\]

\[\Rightarrow A'A=\left[ \begin{matrix}   {{\cos }^{2}}\alpha +{{\sin }^{2}}\alpha  & \sin \alpha \cos \alpha -\sin \alpha \cos \alpha   \\   \sin \alpha \cos \alpha -\sin \alpha \cos \alpha  & {{\cos }^{2}}\alpha +{{\sin }^{2}}\alpha   \\ \end{matrix} \right]\]

\[\therefore A'A=\left[ \begin{matrix}   1 & 0  \\   0 & 1  \\ \end{matrix} \right]\]

\[\therefore A'A=I\]

Hence proved.


7. Show that

i. The matrix \[A=\left[ \begin{matrix}   1 & -1 & 5  \\   -1 & 2 & 1  \\   5 & 1 & 3  \\ \end{matrix} \right]\] is a symmetric matrix.

Ans: Given \[A=\left[ \begin{matrix}   1 & -1 & 5  \\   -1 & 2 & 1  \\   5 & 1 & 3  \\ \end{matrix} \right]\]

\[\therefore A'=\left[ \begin{matrix}   1 & -1 & 5  \\   -1 & 2 & 1  \\   5 & 1 & 3  \\ \end{matrix} \right]\]

Here, we have \[A'=A\]

Thus, \[A\] is a symmetric matrix.

ii. The matrix \[A=\left[ \begin{matrix}   0 & 1 & -1  \\   -1 & 0 & 1  \\   1 & -1 & 0  \\ \end{matrix} \right]\] is a skew symmetric matrix.

Ans: Given \[A=\left[ \begin{matrix}   0 & 1 & -1  \\   -1 & 0 & 1  \\   1 & -1 & 0  \\ \end{matrix} \right]\]

\[\therefore A'=\left[ \begin{matrix}   0 & -1 & 1  \\   1 & 0 & -1  \\   -1 & 1 & 0 \\ \end{matrix} \right]\]

\[\Rightarrow A'=-\left[ \begin{matrix}   0 & 1 & -1  \\   -1 & 0 & 1  \\   1 & -1 & 0  \\ \end{matrix} \right]\]

Here, we have \[A'=-A\]

Thus, \[A\] is a skew symmetric matrix.


8. For the matrix \[A=\left[ \begin{matrix}   1 & 5  \\   6 & 7  \\ \end{matrix} \right]\] , verify that

i. \[\left( A+A' \right)\] is a symmetric matrix.

Ans: Given \[A=\left[ \begin{matrix}   1 & 5  \\   6 & 7  \\ \end{matrix} \right]\]

\[\therefore A'=\left[ \begin{matrix}   1 & 6  \\   5 & 7  \\ \end{matrix} \right]\]

\[A+A'=\left[ \begin{matrix}   1 & 5  \\   6 & 7  \\ \end{matrix} \right]+\left[ \begin{matrix}   1 & 6  \\   5 & 7  \\ \end{matrix} \right]\]

\[\Rightarrow A+A'=\left[ \begin{matrix}   2 & 11  \\   11 & 14  \\ \end{matrix} \right]\]

\[\left( A+A' \right)'=\left[ \begin{matrix}   2 & 11  \\   11 & 14  \\ \end{matrix} \right]\]

Here, we have \[A+A'=\left( A+A' \right)'\]

Thus, \[A+A'\] is a symmetric matrix.

ii. \[\left( A-A' \right)\] is a skew symmetric matrix.

Ans: Given \[A=\left[ \begin{matrix}   1 & 5  \\   6 & 7  \\ \end{matrix} \right]\]

\[\therefore A'=\left[ \begin{matrix}   1 & 6  \\   5 & 7  \\ \end{matrix} \right]\]

\[A-A'=\left[ \begin{matrix}   1 & 5  \\   6 & 7  \\ \end{matrix} \right]-\left[ \begin{matrix}   1 & 6  \\   5 & 7  \\ \end{matrix} \right]\]

\[\Rightarrow A-A'=\left[ \begin{matrix}   0 & -1  \\   1 & 0  \\ \end{matrix} \right]\]

\[\left( A-A' \right)'=\left[ \begin{matrix}   0 & 1  \\   -1 & 0  \\ \end{matrix} \right]\]

\[\Rightarrow \left( A-A' \right)'=-\left[ \begin{matrix}   0 & -1  \\   1 & 0  \\ \end{matrix} \right]\]

Here, we have \[A-A'=-\left( A-A' \right)'\]

Thus, \[A-A'\] is a skew symmetric matrix.


9. Find \[\dfrac{1}{2}\left( A+A' \right)\] and \[\dfrac{1}{2}\left( A-A' \right)\] , when \[A=\left[ \begin{matrix}   0 & a & b  \\   -a & 0 & c  \\   -b & -c & 0  \\ \end{matrix} \right]\] .

Ans: Given \[A=\left[ \begin{matrix}   0 & a & b  \\   -a & 0 & c  \\   -b & -c & 0  \\ \end{matrix} \right]\]

\[\therefore A'=\left[ \begin{matrix}   0 & -a & -b  \\   a & 0 & -c  \\   b & c & 0  \\ \end{matrix} \right]\]

\[A+A'=\left[ \begin{matrix}   0 & a & b  \\   -a & 0 & c  \\   -b & -c & 0  \\ \end{matrix} \right]+\left[ \begin{matrix}   0 & -a & -b  \\   a & 0 & -c  \\   b & c & 0 \\ \end{matrix} \right]\]

\[\Rightarrow A+A'=\left[ \begin{matrix}   0 & 0 & 0  \\   0 & 0 & 0  \\   0 & 0 & 0  \\ \end{matrix} \right]\]

\[\therefore \dfrac{1}{2}\left( A+A' \right)=\left[ \begin{matrix}   0 & 0 & 0  \\   0 & 0 & 0  \\   0 & 0 & 0  \\ \end{matrix} \right]\]

Now, \[A-A'=\left[ \begin{matrix}   0 & a & b  \\   -a & 0 & c  \\   -b & -c & 0  \\ \end{matrix} \right]-\left[ \begin{matrix}   0 & -a & -b  \\   a & 0 & -c  \\   b & c & 0  \\ \end{matrix} \right]\]

\[\left( A-A' \right)=\left[ \begin{matrix}   0 & 2a & 2b  \\  -2a & 0 & 2c  \\   -2b & -2c & 0  \\ \end{matrix} \right]\]

\[\therefore \dfrac{1}{2}\left( A-A' \right)=\left[ \begin{matrix}   0 & a & b  \\   -a & 0 & c  \\   -b & -c & 0  \\ \end{matrix} \right]\]


10. Express the following matrices as the sum of a symmetric and a skew symmetric matrix:

i. \[\left[ \begin{matrix}   3 & 5  \\   1 & -1  \\ \end{matrix} \right]\]

Ans: Let \[A=\left[ \begin{matrix}  3 & 5  \\   1 & -1  \\ \end{matrix} \right]\] and \[A'=\left[ \begin{matrix}   3 & 1  \\   5 & -1  \\ \end{matrix} \right]\]

Now, \[A+A'=\left[ \begin{matrix}   3 & 5  \\   1 & -1  \\ \end{matrix} \right]+\left[ \begin{matrix}   3 & 1  \\   5 & -1  \\ \end{matrix} \right]\]

\[\Rightarrow A+A'=\left[ \begin{matrix}   6 & 6  \\   6 & -2  \\ \end{matrix} \right]\]

Let \[P=\dfrac{1}{2}\left( A+A' \right)\]

\[\Rightarrow P=\dfrac{1}{2}\left[ \begin{matrix}  6 & 6  \\   6 & -2  \\ \end{matrix} \right]\]

\[\Rightarrow P=\left[ \begin{matrix}   3 & 3  \\   3 & -1  \\ \end{matrix} \right]\]

Now, \[P'=\left[ \begin{matrix}   3 & 3  \\   3 & -1  \\ \end{matrix} \right]\]

Here, \[P=P'\]

\[\therefore P=\dfrac{1}{2}\left( A+A' \right)\] is a symmetric matrix.

Now, \[A-A'=\left[ \begin{matrix}   3 & 5  \\   1 & -1  \\ \end{matrix} \right]-\left[ \begin{matrix}   3 & 1  \\   5 & -1  \\ \end{matrix} \right]\]

\[\Rightarrow A-A'=\left[ \begin{matrix}   0 & 4  \\   -4 & 0  \\ \end{matrix} \right]\]

Let \[Q=\dfrac{1}{2}\left( A-A' \right)\]

\[\Rightarrow Q=\dfrac{1}{2}\left[ \begin{matrix}   0 & 4  \\   -4 & 0  \\ \end{matrix} \right]\]

\[\Rightarrow Q=\left[ \begin{matrix}   0 & 2  \\   -2 & 0  \\ \end{matrix} \right]\]

Now, \[Q'=\left[ \begin{matrix}   0 & 2  \\   -2 & 0  \\ \end{matrix} \right]\]

Here, \[Q=-Q'\]

\[\therefore Q=\dfrac{1}{2}\left( A-A' \right)\] is a skew symmetric matrix.

Thus, \[A\]is the sum of matrices \[P=\left[ \begin{matrix}   3 & 3  \\   3 & -1  \\ \end{matrix} \right]\] and \[Q=\left[ \begin{matrix}   0 & 2  \\   -2 & 0  \\ \end{matrix} \right]\].

ii. \[\left[ \begin{matrix}   6 & -2 & 2  \\   -2 & 3 & -1  \\   2 & -1 & 3  \\ \end{matrix} \right]\]

Ans: Let \[A=\left[ \begin{matrix}   6 & -2 & 2  \\   -2 & 3 & -1  \\   2 & -1 & 3  \\ \end{matrix} \right]\] and \[A'=\left[ \begin{matrix}   6 & -2 & 2  \\   -2 & 3 & -1 \\   2 & -1 & 3  \\ \end{matrix} \right]\]

Now, \[A+A'=\left[ \begin{matrix}   6 & -2 & 2  \\   -2 & 3 & -1  \\   2 & -1 & 3  \\ \end{matrix} \right]+\left[ \begin{matrix}   6 & -2 & 2  \\   -2 & 3 & -1  \\   2 & -1 & 3  \\ \end{matrix} \right]\]

Let \[P=\dfrac{1}{2}\left( A+A' \right)\]

\[\Rightarrow P=\dfrac{1}{2}\left[ \begin{matrix}   12 & -4 & 4  \\   -4 & 6 & -2 \\   4 & -2 & 6  \\ \end{matrix} \right]\]

\[\Rightarrow P=\left[ \begin{matrix}   6 & -2 & 2  \\   -2 & 3 & -1  \\   2 & -1 & 3  \\ \end{matrix} \right]\]

Now, \[P'=\left[ \begin{matrix}   6 & -2 & 2  \\   -2 & 3 & -1  \\   2 & -1 & 3  \\ \end{matrix} \right]\]

Here, \[P=P'\]

\[\therefore P=\dfrac{1}{2}\left( A+A' \right)\] is a symmetric matrix.

Now, \[A-A'=\left[ \begin{matrix}  6 & -2 & 2  \\   -2 & 3 & -1  \\   2 & -1 & 3  \\ \end{matrix} \right]-\left[ \begin{matrix}   6 & -2 & 2  \\   -2 & 3 & -1  \\   2 & -1 & 3  \\ \end{matrix} \right]\]

\[\Rightarrow A-A'=\left[ \begin{matrix}   0 & 0 & 0  \\   0 & 0 & 0  \\   0 & 0 & 0  \\ \end{matrix} \right]\]

Let \[Q=\dfrac{1}{2}\left( A-A' \right)\]

\[\Rightarrow Q=\dfrac{1}{2}\left[ \begin{matrix}   0 & 0 & 0  \\   0 & 0 & 0  \\   0 & 0 & 0  \\ \end{matrix} \right]\]

\[\Rightarrow Q=\left[ \begin{matrix}   0 & 0 & 0  \\   0 & 0 & 0  \\   0 & 0 & 0 \\ \end{matrix} \right]\]

Now, \[Q'=\left[ \begin{matrix}   0 & 0 & 0  \\   0 & 0 & 0  \\   0 & 0 & 0  \\ \end{matrix} \right]\]

Here, \[Q=-Q'\]

\[\therefore Q=\dfrac{1}{2}\left( A-A' \right)\] is a skew symmetric matrix.

Thus, \[A\]is the sum of matrices \[P=\left[ \begin{matrix}  6 & -2 & 2  \\   -2 & 3 & -1  \\   2 & -1 & 3  \\ \end{matrix} \right]\] and \[Q=\left[ \begin{matrix}   0 & 0 & 0  \\  0 & 0 & 0  \\   0 & 0 & 0  \\ \end{matrix} \right]\].

iii. \[\left[ \begin{matrix}   3 & 3 & -1  \\   -2 & -2 & 1  \\   -4 & -5 & 2  \\ \end{matrix} \right]\]

Ans: Let \[A=\left[ \begin{matrix}   3 & 3 & -1  \\   -2 & -2 & 1  \\   -4 & -5 & 2  \\ \end{matrix} \right]\] and \[A'=\left[ \begin{matrix}   3 & -2 & -4  \\   3 & -2 & -5  \\   -1 & 1 & 2  \\ \end{matrix} \right]\]

Now, \[A+A'=\left[ \begin{matrix}   3 & 3 & -1  \\   -2 & -2 & 1  \\   -4 & -5 & 2  \\ \end{matrix} \right]+\left[ \begin{matrix}   3 & -2 & -4  \\   3 & -2 & -5  \\   -1 & 1 & 2  \\ \end{matrix} \right]\]

\[\Rightarrow A+A'=\left[ \begin{matrix}   6 & 1 & -5  \\   1 & -4 & -4  \\   -5 & -4 & 4  \\ \end{matrix} \right]\]

Let \[P=\dfrac{1}{2}\left( A+A' \right)\]

\[\Rightarrow P=\dfrac{1}{2}\left[ \begin{matrix}   6 & 1 & -5  \\   1 & -4 & -4 \\   -5 & -4 & 4  \\ \end{matrix} \right]\]

\[\Rightarrow P=\left[ \begin{matrix}   3 & \dfrac{1}{2} & \dfrac{-5}{2}  \\   \dfrac{1}{2} & -2 & -2  \\   \dfrac{-5}{2} & -2 & 2  \\ \end{matrix} \right]\]

Now, \[P'=\left[ \begin{matrix}   3 & \dfrac{1}{2} & \dfrac{-5}{2}  \\  \dfrac{1}{2} & -2 & -2  \\   \dfrac{-5}{2} & -2 & 2  \\ \end{matrix} \right]\]

Here, \[P=P'\]

\[\therefore P=\dfrac{1}{2}\left( A+A' \right)\] is a symmetric matrix.

Now, \[A-A'=\left[ \begin{matrix}   3 & 3 & -1  \\   -2 & -2 & 1  \\   -4 & -5 & 2  \\ \end{matrix} \right]-\left[ \begin{matrix}   3 & -2 & -4  \\   3 & -2 & -5  \\   -1 & 1 & 2  \\ \end{matrix} \right]\]

\[\Rightarrow A-A'=\left[ \begin{matrix}   0 & 5 & 3  \\   -5 & 0 & 6  \\   -3 & -6 & 0  \\ \end{matrix} \right]\]

Let \[Q=\dfrac{1}{2}\left( A-A' \right)\]

\[\Rightarrow Q=\dfrac{1}{2}\left[ \begin{matrix}   0 & 5 & 3  \\   -5 & 0 & 6  \\   -3 & -6 & 0  \\ \end{matrix} \right]\]

\[\Rightarrow Q=\left[ \begin{matrix}   0 & \dfrac{5}{2} & \dfrac{3}{2}  \\   \dfrac{-5}{2} & 0 & 3  \\   \dfrac{-3}{2} & -3 & 0  \\ \end{matrix} \right]\]

Now, \[Q'=\left[ \begin{matrix}   0 & -\dfrac{5}{2} & -\dfrac{3}{2}  \\ \dfrac{5}{2} & 0 & -3  \\   \dfrac{3}{2} & 3 & 0  \\ \end{matrix} \right]\]

Here, \[Q=-Q'\]

\[\therefore Q=\dfrac{1}{2}\left( A-A' \right)\] is a skew symmetric matrix.

Thus, \[A\]is the sum of matrices \[P=\left[ \begin{matrix}   3 & \dfrac{1}{2} & \dfrac{-5}{2}  \\   \dfrac{1}{2} & -2 & -2  \\  \dfrac{-5}{2} & -2 & 2  \\ \end{matrix} \right]\] and \[Q=\left[ \begin{matrix}  0 & \dfrac{5}{2} & \dfrac{3}{2}  \\    \dfrac{-5}{2} & 0 & 3  \\  \dfrac{-3}{2} & -3 & 0  \\ \end{matrix} \right]\].

iv. \[\left[ \begin{matrix}   1 & 5  \\   -1 & 2  \\ \end{matrix} \right]\]

Ans: Let \[A=\left[ \begin{matrix}   1 & 5  \\   -1 & 2  \\ \end{matrix} \right]\] and \[A'=\left[ \begin{matrix}   1 & -1  \\   5 & 2  \\ \end{matrix} \right]\]

Now, \[A+A'=\left[ \begin{matrix}   1 & 5  \\   -1 & 2  \\ \end{matrix} \right]+\left[ \begin{matrix}   1 & -1  \\   5 & 2  \\ \end{matrix} \right]\]

\[\Rightarrow A+A'=\left[ \begin{matrix}   2 & 4  \\   4 & 4  \\ \end{matrix} \right]\]

Let \[P=\dfrac{1}{2}\left( A+A' \right)\]

\[\Rightarrow P=\dfrac{1}{2}\left[ \begin{matrix}   2 & 4  \\   4 & 4  \\ \end{matrix} \right]\]

\[\Rightarrow P=\left[ \begin{matrix}   1 & 2  \\   2 & 2  \\ \end{matrix} \right]\]

Now, \[P'=\left[ \begin{matrix}   1 & 2  \\   2 & 2  \\ \end{matrix} \right]\]

Here, \[P=P'\]

\[\therefore P=\dfrac{1}{2}\left( A+A' \right)\] is a symmetric matrix.

Now, \[A-A'=\left[ \begin{matrix}   1 & 5  \\   -1 & 2  \\ \end{matrix} \right]-\left[ \begin{matrix}   1 & -1  \\   5 & 2  \\ \end{matrix} \right]\]

\[\Rightarrow A-A'=\left[ \begin{matrix}   0 & 6  \\   -6 & 0  \\ \end{matrix} \right]\]

Let \[Q=\dfrac{1}{2}\left( A-A' \right)\]

\[\Rightarrow Q=\dfrac{1}{2}\left[ \begin{matrix}   0 & 6  \\   -6 & 0  \\ \end{matrix} \right]\]

\[\Rightarrow Q=\left[ \begin{matrix}   0 & 3  \\   -3 & 0  \\ \end{matrix} \right]\]

Now, \[Q'=\left[ \begin{matrix}    0 & -3  \\   3 & 0  \\ \end{matrix} \right]\]

Here, \[Q=-Q'\]

\[\therefore Q=\dfrac{1}{2}\left( A-A' \right)\] is a skew symmetric matrix.

Thus, \[A\]is the sum of matrices \[P=\left[ \begin{matrix}   1 & 2  \\   2 & 2  \\ \end{matrix} \right]\] and \[Q=\left[ \begin{matrix}   0 & 3  \\   -3 & 0  \\ \end{matrix} \right]\].


11. If \[A,B\] are symmetric matrices of same order, then \[AB-BA\] is a:

  1. Skew symmetric matrix

  2. Symmetric matrix

  3. Zero matrix

  4. Identity matrix

Ans: \[A\] and \[B\] are symmetric matrix, therefore, we have:

\[A'=A\] and \[B'=B\]                                                    …(1)

Here, \[\left( AB-BA \right)'=\left( AB \right)'-\left( BA \right)'\]

\[\Rightarrow \left( AB-BA \right)'=B'A'-A'B'\]

From (1),

\[\Rightarrow \left( AB-BA \right)'=BA-AB\]

\[\Rightarrow \left( AB-BA \right)'=-\left( AB-BA \right)\]                           …(2)

From equation (2),

\[AB-BA\] is a skew symmetric matrix.


12. If \[A=\left[ \begin{matrix}   \cos \alpha  & -\sin \alpha   \\   \sin \alpha  & \cos \alpha   \\ \end{matrix} \right]\] , then \[\mathbf{A+A'=I}\] , if the value of \[\mathbf{\alpha}\] is

  1. \[\dfrac{\pi }{6}\]

  2. \[\dfrac{\pi }{3}\]

  3. \[n\]

  4. \[\dfrac{3\pi }{2}\]

Ans: Given \[A=\left[ \begin{matrix}   \cos \alpha  & -\sin \alpha   \\   \sin \alpha  & \cos \alpha   \\ \end{matrix} \right]\]

\[\Rightarrow A'=\left[ \begin{matrix}   \cos \alpha  & \sin \alpha   \\   -\sin \alpha  & \cos \alpha   \\ \end{matrix} \right]\]

Now, \[\because A+A'=I\]

\[\therefore \left[ \begin{matrix}   \cos \alpha  & -\sin \alpha   \\   \sin \alpha  & \cos \alpha   \\ \end{matrix} \right]+\left[ \begin{matrix}   \cos \alpha  & \sin \alpha   \\    -\sin \alpha  & \cos \alpha   \\ \end{matrix} \right]=\left[ \begin{matrix}   1 & 0  \\   0 & 1  \\ \end{matrix} \right]\]

\[\Rightarrow \left[ \begin{matrix}   2\cos \alpha  & 0  \\   0 & 2\cos \alpha   \\ \end{matrix} \right]=\left[ \begin{matrix}   1 & 0  \\   0 & 1  \\ \end{matrix} \right]\]

Equating the corresponding elements of the two matrices, we have:

\[\cos \alpha =\dfrac{1}{2}\]

\[\Rightarrow \alpha ={{\cos }^{-1}}\left( \dfrac{1}{2} \right)\]

\[\Rightarrow \alpha =\dfrac{\pi }{3}\]

Thus, option (B) is correct.


Exercise 3.4 (Not in the current syllabus)

1. Find the inverse of each of the matrices, if it exists.

$\left[ \begin{matrix}   1 & -1  \\   2 & 3  \\ \end{matrix} \right]$

Ans: Let $A=\left[ \begin{matrix}   1 & -1  \\   2 & 3  \\ \end{matrix} \right]$

Since $A=IA$

$\therefore \left[ \begin{matrix}   1 & -1  \\   0 & 5  \\ \end{matrix} \right]=\left[ \begin{matrix}   1 & 0  \\   0 & 1  \\ \end{matrix} \right]A$

Applying ${{R}_{2}}\to {{R}_{2}}-2{{R}_{1}}$

$\Rightarrow \left[ \begin{matrix}   1 & -1  \\   0 & 5  \\ \end{matrix} \right]=\left[ \begin{matrix}   1 & 0  \\   -2 & 1  \\ \end{matrix} \right]A$

Applying ${{R}_{2}}\to \dfrac{1}{5}{{R}_{2}}$

$\Rightarrow \left[ \begin{matrix}   1 & -1  \\   0 & 5  \\ \end{matrix} \right]=\left[ \begin{matrix}   1 & 0  \\   \dfrac{-2}{5} & \dfrac{1}{5}  \\ \end{matrix} \right]A$

Applying ${{R}_{1}}\to {{R}_{1}}+{{R}_{2}}$

$\Rightarrow \left[ \begin{matrix}   1 & -1  \\   0 & 5  \\ \end{matrix} \right]=\left[ \begin{matrix}   \dfrac{3}{5} & \dfrac{1}{5}  \\   \dfrac{-2}{5} & \dfrac{1}{5}  \\ \end{matrix} \right]A$

$\therefore {{A}^{-1}}=\left[ \begin{matrix}   \dfrac{3}{5} & \dfrac{1}{5}  \\   \dfrac{-2}{5} & \dfrac{1}{5}  \\ \end{matrix} \right]$

2. Find the inverse of each of the matrices, if it exists.

$\left[ \begin{matrix}   2 & 1  \\   1 & 1  \\ \end{matrix} \right]$

Ans: Let $A=\left[ \begin{matrix}   2 & 1  \\   1 & 1  \\ \end{matrix} \right]$

Since $A=IA$

$\therefore \left[ \begin{matrix}   2 & 1  \\   1 & 1  \\ \end{matrix} \right]=\left[ \begin{matrix}   1 & 0  \\   0 & 1  \\ \end{matrix} \right]A$

Applying ${{R}_{1}}\to {{R}_{1}}-{{R}_{2}}$

$\Rightarrow \left[ \begin{matrix}   1 & 0  \\   0 & 1  \\ \end{matrix} \right]=\left[ \begin{matrix}   1 & -1  \\   0 & 1  \\ \end{matrix} \right]A$

Applying ${{R}_{2}}\to {{R}_{2}}-{{R}_{1}}$

$\Rightarrow \left[ \begin{matrix}   1 & -1  \\   0 & 5  \\ \end{matrix} \right]=\left[ \begin{matrix}   1 & -1  \\   -1 & 2  \\ \end{matrix} \right]A$

$\therefore {{A}^{-1}}=\left[ \begin{matrix}   1 & -1  \\   -1 & 2  \\ \end{matrix} \right]$

3. Find the inverse of each of the matrices, if it exists.

$\left[ \begin{matrix}   1 & 3  \\   2 & 7  \\ \end{matrix} \right]$

Ans: Let $A=\left[ \begin{matrix}   1 & 3  \\   2 & 7  \\ \end{matrix} \right]$

Since $A=IA$

$\therefore \left[ \begin{matrix}   1 & 3  \\   2 & 7  \\ \end{matrix} \right]=\left[ \begin{matrix}   1 & 0  \\   0 & 1  \\ \end{matrix} \right]A$

Applying ${{R}_{2}}\to {{R}_{2}}-2{{R}_{1}}$

$\Rightarrow \left[ \begin{matrix}   1 & 3  \\   0 & 1  \\ \end{matrix} \right]=\left[ \begin{matrix}   1 & 0  \\   -2 & 1  \\ \end{matrix} \right]A$

Applying ${{R}_{1}}\to {{R}_{1}}-3{{R}_{2}}$

$\Rightarrow \left[ \begin{matrix}   1 & 0  \\   0 & 1  \\ \end{matrix} \right]=\left[ \begin{matrix}   7 & -3  \\   -2 & 1  \\ \end{matrix} \right]A$

$\therefore {{A}^{-1}}=\left[ \begin{matrix}   7 & -3  \\   -2 & 1  \\ \end{matrix} \right]$

4. Find the inverse of each of the matrices, if it exists.

$\left[ \begin{matrix}   2 & 3  \\  5 & 7  \\ \end{matrix} \right]$

Ans: Let $A=\left[ \begin{matrix}   2 & 3  \\   5 & 7  \\ \end{matrix} \right]$

Since $A=IA$

$\therefore \left[ \begin{matrix}   2 & 3  \\  5 & 7  \\ \end{matrix} \right]=\left[ \begin{matrix}   1 & 0  \\   0 & 1  \\ \end{matrix} \right]A$

Applying ${{R}_{2}}\to {{R}_{2}}-2{{R}_{1}}$

$\Rightarrow \left[ \begin{matrix}   2 & 3  \\   1 & 1  \\ \end{matrix} \right]=\left[ \begin{matrix}   1 & 0  \\   -2 & 1  \\ \end{matrix} \right]A$

Applying ${{R}_{1}}\leftrightarrow {{R}_{2}}$

$\Rightarrow \left[ \begin{matrix}   1 & 1  \\   2 & 3  \\ \end{matrix} \right]=\left[ \begin{matrix}   -2 & 1  \\   1 & 0  \\ \end{matrix} \right]A$

Applying ${{R}_{2}}\leftrightarrow {{R}_{2}}-2{{R}_{1}}$

$\Rightarrow \left[ \begin{matrix}   1 & 1  \\   0 & 1  \\ \end{matrix} \right]=\left[ \begin{matrix}   -2 & 1  \\   5 & -2  \\ \end{matrix} \right]A$

Applying ${{R}_{1}}\to {{R}_{1}}-{{R}_{2}}$

$\Rightarrow \left[ \begin{matrix}   1 & 0  \\   0 & 1  \\ \end{matrix} \right]=\left[ \begin{matrix}   -7 & 3  \\   5 & -2  \\ \end{matrix} \right]A$

$\therefore {{A}^{-1}}=\left[ \begin{matrix}   7 & 3  \\   5 & -2  \\ \end{matrix} \right]$

5. Find the inverse of each of the matrices, if it exists.

$\left[ \begin{matrix}   2 & 1  \\   7 & 4  \\ \end{matrix} \right]$

Ans: Let $A=\left[ \begin{matrix}   2 & 1  \\   7 & 4  \\ \end{matrix} \right]$

Since $A=IA$

$\therefore \left[ \begin{matrix}   2 & 1  \\   7 & 4  \\ \end{matrix} \right]=\left[ \begin{matrix}   1 & 0  \\   0 & 1  \\ \end{matrix} \right]A$

Applying ${{R}_{2}}\to {{R}_{2}}-3{{R}_{1}}$

$\Rightarrow \left[ \begin{matrix}   2 & 1  \\   1 & 1  \\ \end{matrix} \right]=\left[ \begin{matrix}   1 & 0  \\   -3 & 1  \\ \end{matrix} \right]A$

Applying ${{R}_{1}}\to {{R}_{1}}-{{R}_{2}}$

$\Rightarrow \left[ \begin{matrix}   1 & 0  \\   0 & 1  \\ \end{matrix} \right]=\left[ \begin{matrix}   4 & -1  \\   -7 & 2  \\ \end{matrix} \right]A$

$\therefore {{A}^{-1}}=\left[ \begin{matrix}   4 & -1  \\   -7 & 2  \\ \end{matrix} \right]$

6. Find the inverse of each of the matrices, if it exists.

$\left[ \begin{matrix}  2 & 5  \\   1 & 3  \\ \end{matrix} \right]$

Ans: Let $A=\left[ \begin{matrix}  2 & 5  \\   1 & 3  \\ \end{matrix} \right]$

Since $A=IA$

$\therefore \left[ \begin{matrix}   2 & 5  \\  1 & 3  \\ \end{matrix} \right]=\left[ \begin{matrix}   1 & 0  \\   0 & 1  \\ \end{matrix} \right]A$

Applying ${{R}_{1}}\to \dfrac{1}{2}{{R}_{1}}$

$\Rightarrow \left[ \begin{matrix}   1 & \dfrac{5}{2}  \\  0 & 3  \\ \end{matrix} \right]=\left[ \begin{matrix}   \dfrac{1}{2} & 0  \\   0 & 1  \\ \end{matrix} \right]A$

Applying ${{R}_{2}}\to {{R}_{2}}-{{R}_{1}}$

$\Rightarrow \left[ \begin{matrix}  1 & \dfrac{5}{2}  \\   0 & \dfrac{1}{2}  \\ \end{matrix} \right]=\left[ \begin{matrix}   \dfrac{1}{2} & 0  \\   \dfrac{-1}{2} & 1 \\ \end{matrix} \right]A$

Applying ${{R}_{1}}\to {{R}_{1}}-5{{R}_{2}}$

$\Rightarrow \left[ \begin{matrix}   1 & 0  \\   0 & \dfrac{1}{2}  \\ \end{matrix} \right]=\left[ \begin{matrix}   3 & -5  \\   \dfrac{-1}{2} & 1  \\ \end{matrix} \right]A$

Applying ${{R}_{2}}\to 2{{R}_{2}}$

$\Rightarrow \left[ \begin{matrix}   1 & 0  \\   0 & 1  \\ \end{matrix} \right]=\left[ \begin{matrix}   3 & -5  \\   -1 & 2  \\ \end{matrix} \right]A$

$\therefore {{A}^{-1}}=\left[ \begin{matrix}   3 & -5  \\   -1 & 2  \\ \end{matrix} \right]$

7. Find the inverse of each of the matrices, if it exists.

$\left[ \begin{matrix}   3 & 1  \\   5 & 2  \\ \end{matrix} \right]$

Ans: Let $A=\left[ \begin{matrix}   3 & 1  \\   5 & 2  \\ \end{matrix} \right]$

Since $A=IA$

$\therefore \left[ \begin{matrix}   3 & 1  \\   5 & 2  \\ \end{matrix} \right]=\left[ \begin{matrix}   1 & 0  \\   0 & 1  \\ \end{matrix} \right]A$

Applying ${{R}_{1}}\to 2{{R}_{1}}$

$\Rightarrow \left[ \begin{matrix}   6 & 2  \\   5 & 2  \\ \end{matrix} \right]=\left[ \begin{matrix}   2 & 0  \\   0 & 1  \\ \end{matrix} \right]A$

Applying ${{R}_{1}}\to {{R}_{1}}-{{R}_{2}}$

$\Rightarrow \left[ \begin{matrix}   1 & 0  \\   5 & 2  \\ \end{matrix} \right]=\left[ \begin{matrix}   2 & -1  \\   0 & 1  \\ \end{matrix} \right]A$

Applying ${{R}_{2}}\to {{R}_{2}}-5{{R}_{1}}$

$\Rightarrow \left[ \begin{matrix}   1 & 0  \\   0 & 2  \\ \end{matrix} \right]=\left[ \begin{matrix}   2 & -1  \\   -10 & 6  \\ \end{matrix} \right]A$

Applying ${{R}_{2}}\to \dfrac{1}{2}{{R}_{2}}$

$\Rightarrow \left[ \begin{matrix}   1 & 0  \\   0 & 1  \\ \end{matrix} \right]=\left[ \begin{matrix}   2 & -1  \\  -5 & 3  \\ \end{matrix} \right]A$

$\therefore {{A}^{-1}}=\left[ \begin{matrix}   2 & -1  \\   -5 & 3  \\ \end{matrix} \right]$

8. Find the inverse of each of the matrices, if it exists.

$\left[ \begin{matrix}   4 & 5  \\   3 & 4  \\ \end{matrix} \right]$

Ans: Let $A=\left[ \begin{matrix}   4 & 5  \\   3 & 4  \\ \end{matrix} \right]$

Since $A=IA$

$\therefore \left[ \begin{matrix}   4 & 5  \\  3 & 4  \\ \end{matrix} \right]=\left[ \begin{matrix}  1 & 0  \\  0 & 1  \\ \end{matrix} \right]A$

Applying ${{R}_{1}}\to {{R}_{1}}-{{R}_{2}}$

$\Rightarrow \left[ \begin{matrix}   1 & 1  \\   3 & 4  \\ \end{matrix} \right]=\left[ \begin{matrix}   1 & -1  \\   0 & 1  \\ \end{matrix} \right]A$

Applying ${{R}_{2}}\to {{R}_{2}}-3{{R}_{1}}$

$\Rightarrow \left[ \begin{matrix}   1 & 1  \\   0 & 1  \\ \end{matrix} \right]=\left[ \begin{matrix}   4 & -5  \\   -3 & 4  \\ \end{matrix} \right]A$

Applying ${{R}_{1}}\to {{R}_{1}}-{{R}_{2}}$

$\Rightarrow \left[ \begin{matrix}   1 & 0  \\   0 & 1  \\ \end{matrix} \right]=\left[ \begin{matrix}   4 & -5  \\   -3 & 4  \\ \end{matrix} \right]A$

$\therefore {{A}^{-1}}=\left[ \begin{matrix}   4 & -5  \\   -3 & 4  \\ \end{matrix} \right]$

9. Find the inverse of each of the matrices, if it exists.

$\left[ \begin{matrix}   3 & 10  \\   2 & 7  \\ \end{matrix} \right]$

Ans: Let $A=\left[ \begin{matrix}   3 & 10  \\   2 & 7  \\ \end{matrix} \right]$

Since $A=IA$

$\therefore \left[ \begin{matrix}   3 & 10  \\   2 & 7  \\ \end{matrix} \right]=\left[ \begin{matrix}   1 & 0  \\  0 & 1  \\ \end{matrix} \right]A$

Applying ${{R}_{1}}\to {{R}_{1}}-{{R}_{2}}$

$\Rightarrow \left[ \begin{matrix}   1 & 3  \\   2 & 7  \\ \end{matrix} \right]=\left[ \begin{matrix}   1 & -1  \\   0 & 1  \\ \end{matrix} \right]A$

Applying ${{R}_{2}}\to {{R}_{2}}-2{{R}_{1}}$

$\Rightarrow \left[ \begin{matrix}   1 & 3  \\  0 & 1  \\ \end{matrix} \right]=\left[ \begin{matrix}   1 & -1  \\  -2 & 3  \\ \end{matrix} \right]A$

Applying ${{R}_{1}}\to {{R}_{1}}-3{{R}_{2}}$

$\Rightarrow \left[ \begin{matrix}   1 & 0  \\  0 & 1  \\ \end{matrix} \right]=\left[ \begin{matrix}   7 & -10  \\   -2 & 3  \\ \end{matrix} \right]A$

$\therefore {{A}^{-1}}=\left[ \begin{matrix}  7 & -10  \\   -2 & 3  \\ \end{matrix} \right]$

10. Find the inverse of each of the matrices, if it exists.

$\left[ \begin{matrix}   3 & -1  \\   -4 & 2  \\ \end{matrix} \right]$

Ans: Let $A=\left[ \begin{matrix}   3 & -1  \\   -4 & 2  \\ \end{matrix} \right]$

Since $A=IA$

$\therefore \left[ \begin{matrix}   3 & -1  \\   -4 & 2  \\ \end{matrix} \right]=\left[ \begin{matrix}  1 & 0  \\  0 & 1  \\ \end{matrix} \right]A$

Applying ${{R}_{1}}\to {{R}_{1}}+{{R}_{2}}$

$\Rightarrow \left[ \begin{matrix}   1 & -1  \\  -4 & 2  \\ \end{matrix} \right]=\left[ \begin{matrix}    1 & 1  \\   0 & 1  \\ \end{matrix} \right]A$

Applying ${{R}_{1}}\to -{{R}_{1}}$

$\Rightarrow \left[ \begin{matrix}   1 & -1  \\   0 & -2  \\ \end{matrix} \right]=\left[ \begin{matrix}   -1 & -1  \\   0 & 1  \\ \end{matrix} \right]A$

Applying ${{R}_{2}}\to {{R}_{2}}+4{{R}_{1}}$

$\Rightarrow \left[ \begin{matrix}   1 & -1  \\   0 & -2  \\ \end{matrix} \right]=\left[ \begin{matrix}   -1 & -1  \\   -4 & -3  \\ \end{matrix} \right]A$

Applying ${{R}_{2}}\to -\dfrac{1}{2}{{R}_{2}}$

$\Rightarrow \left[ \begin{matrix}   1 & -1  \\   0 & 1  \\ \end{matrix} \right]=\left[ \begin{matrix}   -1 & -1  \\   2 & \dfrac{3}{2}  \\ \end{matrix} \right]A$

Applying ${{R}_{1}}\to {{R}_{1}}+{{R}_{2}}$

$\Rightarrow \left[ \begin{matrix}   1 & 0  \\   0 & 1  \\ \end{matrix} \right]=\left[ \begin{matrix}   1 & \dfrac{1}{2}  \\   2 & \dfrac{3}{2}  \\ \end{matrix} \right]A$

$\therefore {{A}^{-1}}=\left[ \begin{matrix}   1 & \dfrac{1}{2}  \\   2 & \dfrac{3}{2}  \\ \end{matrix} \right]$

11. Find the inverse of each of the matrices, if it exists.

$\left[ \begin{matrix}   2 & -6  \\   1 & -2  \\ \end{matrix} \right]$

Ans: Let $A=\left[ \begin{matrix}   2 & -6  \\   1 & -2  \\ \end{matrix} \right]$

Since $A=IA$

$\therefore \left[ \begin{matrix}   2 & -6  \\  1 & -2  \\ \end{matrix} \right]=\left[ \begin{matrix}   1 & 0  \\   0 & 1  \\ \end{matrix} \right]A$

Applying ${{R}_{1}}\leftrightarrow {{R}_{2}}$

$\Rightarrow \left[ \begin{matrix}   1 & -2  \\   2 & -6  \\ \end{matrix} \right]=\left[ \begin{matrix}   0 & 1  \\   1 & 0  \\ \end{matrix} \right]A$

Applying ${{R}_{2}}\to {{R}_{2}}-2{{R}_{1}}$

$\Rightarrow \left[ \begin{matrix}   1 & -2  \\   0 & -2  \\ \end{matrix} \right]=\left[ \begin{matrix}   0 & 1  \\   1 & -2  \\ \end{matrix} \right]A$

Applying ${{R}_{2}}\to -\dfrac{1}{2}{{R}_{2}}$

$\Rightarrow \left[ \begin{matrix}   1 & -2  \\   0 & 1  \\ \end{matrix} \right]=\left[ \begin{matrix}   0 & 1  \\   \dfrac{-1}{2} & 1  \\ \end{matrix} \right]A$

Applying ${{R}_{1}}\to {{R}_{1}}+2{{R}_{2}}$

$\Rightarrow \left[ \begin{matrix}   1 & 0  \\   0 & 1  \\ \end{matrix} \right]=\left[ \begin{matrix}   -1 & 3  \\   \dfrac{-1}{2} & 1  \\ \end{matrix} \right]A$

$\therefore {{A}^{-1}}=\left[ \begin{matrix}   -1 & 3  \\   \dfrac{-1}{2} & 1  \\ \end{matrix} \right]$

12. Find the inverse of each of the matrices, if it exists.

$\left[ \begin{matrix}   6 & -3  \\   -2 & 1  \\ \end{matrix} \right]$

Ans: Let $A=\left[ \begin{matrix}   6 & -3  \\   -2 & 1  \\ \end{matrix} \right]$

Since $A=IA$

$\therefore \left[ \begin{matrix}   6 & -3  \\   -2 & 1  \\ \end{matrix} \right]=\left[ \begin{matrix}   1 & 0  \\   0 & 1  \\ \end{matrix} \right]A$

Applying ${{R}_{1}}\to \dfrac{1}{6}{{R}_{1}}$

$\Rightarrow \left[ \begin{matrix}   1 & \dfrac{-1}{2}  \\  -2 & 1  \\ \end{matrix} \right]=\left[ \begin{matrix}   \dfrac{1}{6} & 0  \\   0 & 1  \\ \end{matrix} \right]A$

Applying ${{R}_{2}}\to {{R}_{2}}+2{{R}_{1}}$

$\Rightarrow \left[ \begin{matrix}   1 & -\dfrac{1}{2}  \\   0 & 0  \\ \end{matrix} \right]=\left[ \begin{matrix}   \dfrac{1}{6} & 3  \\   \dfrac{1}{3} & 1  \\ \end{matrix} \right]A$

Since, we can see all the zeros in the second row of the matrix on the L.H.S, $\therefore {{A}^{-1}}$ does not exist.

13. Find the inverse of each of the matrices, if it exists.

$\left[ \begin{matrix}  2 & -3  \\   -1 & 2  \\ \end{matrix} \right]$

Ans: Let $A=\left[ \begin{matrix}   2 & -3  \\   -1 & 2  \\ \end{matrix} \right]$

Since $A=IA$

$\therefore \left[ \begin{matrix}   2 & -3  \\  -1 & 2  \\ \end{matrix} \right]=\left[ \begin{matrix}  1 & 0  \\   0 & 1  \\ \end{matrix} \right]A$

Applying ${{R}_{1}}\to {{R}_{1}}+{{R}_{2}}$

$\Rightarrow \left[ \begin{matrix}   1 & -1  \\  -1 & 2  \\ \end{matrix} \right]=\left[ \begin{matrix}   1 & 1  \\   0 & 1  \\ \end{matrix} \right]A$

Applying ${{R}_{2}}\to {{R}_{2}}+{{R}_{1}}$

$\Rightarrow \left[ \begin{matrix}   1 & -1  \\   0 & 1  \\ \end{matrix} \right]=\left[ \begin{matrix}   1 & 1  \\   1 & 2  \\ \end{matrix} \right]A$

Applying ${{R}_{1}}\to {{R}_{1}}+{{R}_{2}}$

$\Rightarrow \left[ \begin{matrix}   1 & 0  \\   0 & 1  \\ \end{matrix} \right]=\left[ \begin{matrix}   2 & 3  \\   1 & 2  \\ \end{matrix} \right]A$

$\therefore {{A}^{-1}}=\left[ \begin{matrix}   2 & 3  \\   1 & 2  \\ \end{matrix} \right]$

14. Find the inverse of each of the matrices, if it exists.

$\left[ \begin{matrix}   2 & 1  \\   4 & 2  \\ \end{matrix} \right]$

Ans: Let $A=\left[ \begin{matrix}   2 & 1  \\   4 & 2  \\ \end{matrix} \right]$

Since $A=IA$

$\therefore \left[ \begin{matrix}   2 & 1  \\   4 & 2  \\ \end{matrix} \right]=\left[ \begin{matrix}   1 & 0  \\   0 & 1  \\ \end{matrix} \right]A$

Applying ${{R}_{1}}\to {{R}_{1}}-\dfrac{1}{2}{{R}_{2}}$

$\Rightarrow \left[ \begin{matrix}   0 & 0  \\   4 & 2  \\ \end{matrix} \right]=\left[ \begin{matrix}   1 & \dfrac{-1}{2}  \\   0 & 1  \\ \end{matrix} \right]A$

Since, we can see all the zeros in the first row of the matrix on the L.H.S, $\therefore {{A}^{-1}}$ does not exist.

15.  Find the inverse of each of the matrices, if it exists.

$\left[ \begin{matrix}   1 & 3 & -2  \\   -3 & 0 & -5  \\   2 & 5 & 0  \\ \end{matrix} \right]$

Ans: Let $A=\left[ \begin{matrix}   1 & 3 & -2  \\   -3 & 0 & -5  \\   2 & 5 & 0  \\ \end{matrix} \right]$

Since $A=IA$

$\therefore \left[ \begin{matrix}   1 & 3 & -2  \\   -3 & 0 & -5  \\   2 & 5 & 0  \\ \end{matrix} \right]=\left[ \begin{matrix}   1 & 0 & 0  \\   0 & 1 & 0  \\   0 & 0 & 1 \\ \end{matrix} \right]A$

Applying ${{R}_{2}}\to {{R}_{2}}+3{{R}_{1}}$ and ${{R}_{3}}\to {{R}_{3}}-2{{R}_{1}}$

$\Rightarrow \left[ \begin{matrix}   1 & 3 & -2  \\   0 & 9 & -11  \\   0 & -1 & 4  \\ \end{matrix} \right]=\left[ \begin{matrix}   1 & 0 & 0  \\   3 & 1 & 0  \\   -2 & 0 & 1  \\ \end{matrix} \right]A$

Applying ${{R}_{1}}\to {{R}_{1}}+3{{R}_{3}}$ and ${{R}_{2}}\to {{R}_{2}}+8{{R}_{3}}$

$\Rightarrow \left[ \begin{matrix}   1 & 0 & 10  \\   0 & 1 & 21  \\   0 & -1 & 4  \\ \end{matrix} \right]=\left[ \begin{matrix}   -5 & 0 & 3  \\   -13 & 1 & 8  \\   -2 & 0 & 1  \\ \end{matrix} \right]A$

Applying ${{R}_{3}}\to {{R}_{3}}+{{R}_{2}}$

$\Rightarrow \left[ \begin{matrix}   1 & 0 & 10  \\   0 & 1 & 21  \\   0 & 0 & 25  \\ \end{matrix} \right]=\left[ \begin{matrix}   -5 & 0 & 3  \\   -13 & 1 & 8  \\   -15 & 1 & 9  \\ \end{matrix} \right]A$

Applying ${{R}_{3}}\to \dfrac{1}{25}{{R}_{3}}$

$\Rightarrow \left[ \begin{matrix}   1 & 0 & 10  \\   0 & 1 & 21  \\   0 & 0 & 1  \\ \end{matrix} \right]=\left[ \begin{matrix}   -5 & 0 & 3  \\   -13 & 1 & 8  \\   -\dfrac{3}{5} & \dfrac{1}{25} & \dfrac{9}{25}  \\ \end{matrix} \right]A$

Applying ${{R}_{1}}\to {{R}_{1}}-10{{R}_{3}}$ and ${{R}_{2}}\to {{R}_{2}}-21{{R}_{3}}$

$\Rightarrow \left[ \begin{matrix}   1 & 0 & 0  \\   0 & 1 & 0  \\   0 & 0 & 1  \\ \end{matrix} \right]=\left[ \begin{matrix}   1 & -\dfrac{2}{5} & -\dfrac{3}{5}  \\  -\dfrac{2}{5} & \dfrac{4}{25} & \dfrac{11}{25}  \\   -\dfrac{3}{5} & \dfrac{1}{25} & \dfrac{9}{25}  \\ \end{matrix} \right]A$

$\therefore {{A}^{-1}}=\left[ \begin{matrix}   1 & -\dfrac{2}{5} & -\dfrac{3}{5}  \\   -\dfrac{2}{5} & \dfrac{4}{25} & \dfrac{11}{25}  \\   -\dfrac{3}{5} & \dfrac{1}{25} & \dfrac{9}{25}  \\ \end{matrix} \right]$

16. Find the inverse of each of the matrices, if it exists.

$\left[ \begin{matrix}   2 & 0 & -1  \\   5 & 1 & 0  \\   0 & 1 & 3  \\ \end{matrix} \right]$

Ans: Let $A=\left[ \begin{matrix}   2 & 0 & -1  \\   5 & 1 & 0  \\   0 & 1 & 3  \\ \end{matrix} \right]$

Since $A=IA$

$\therefore \left[ \begin{matrix}   2 & 0 & -1  \\  5 & 1 & 0  \\   0 & 1 & 3  \\ \end{matrix} \right]=\left[ \begin{matrix}   1 & 0 & 0  \\   0 & 1 & 0  \\   0 & 0 & 1 \\ \end{matrix} \right]A$

Applying ${{R}_{1}}\to \dfrac{1}{2}{{R}_{1}}$

$\Rightarrow \left[ \begin{matrix}   1 & 0 & -\dfrac{1}{2}  \\   5 & 1 & 0  \\   0 & 1 & 3  \\ \end{matrix} \right]=\left[ \begin{matrix}   \dfrac{1}{2} & 0 & 0  \\   0 & 1 & 0  \\   0 & 0 & 1  \\ \end{matrix} \right]A$

Applying ${{R}_{2}}\to {{R}_{2}}-5{{R}_{1}}$

$\Rightarrow \left[ \begin{matrix}   1 & 0 & -\dfrac{1}{2}  \\   0 & 1 & \dfrac{5}{2}  \\   0 & 1 & 3  \\ \end{matrix} \right]=\left[ \begin{matrix}   \dfrac{1}{2} & 0 & 0  \\   -\dfrac{5}{2} & 1 & 0  \\   0 & 0 & 1  \\ \end{matrix} \right]A$

Applying ${{R}_{3}}\to {{R}_{3}}-{{R}_{2}}$

$\Rightarrow \left[ \begin{matrix}   1 & 0 & -\dfrac{1}{2}  \\   0 & 1 & \dfrac{5}{2} \\   0 & 0 & \dfrac{1}{2}  \\ \end{matrix} \right]=\left[ \begin{matrix}   \dfrac{1}{2} & 0 & 0  \\   -\dfrac{5}{2} & 1 & 0  \\   \dfrac{5}{2} & -1 & 1  \\ \end{matrix} \right]A$

Applying ${{R}_{3}}\to 2{{R}_{3}}$

$\Rightarrow \left[ \begin{matrix}   1 & 0 & -\dfrac{1}{2}  \\   0 & 1 & \dfrac{5}{2} \\   0 & 0 & 1  \\ \end{matrix} \right]=\left[ \begin{matrix}   \dfrac{1}{2} & 0 & 0  \\   -\dfrac{5}{2} & 1 & 0  \\   5 & -2 & 2  \\ \end{matrix} \right]A$

Applying ${{R}_{1}}\to {{R}_{1}}+\dfrac{1}{2}{{R}_{3}}$ and ${{R}_{2}}\to {{R}_{2}}-\dfrac{5}{2}{{R}_{3}}$

$\Rightarrow \left[ \begin{matrix}   1 & 0 & 0  \\   0 & 1 & 0  \\   0 & 0 & 1  \\ \end{matrix} \right]=\left[ \begin{matrix}   3 & -1 & 1  \\   -15 & 6 & -5  \\   5 & -2 & 2  \\ \end{matrix} \right]A$

$\therefore {{A}^{-1}}=\left[ \begin{matrix}   3 & -1 & 1  \\   -15 & 6 & -5  \\   5 &-2 & 2  \\ \end{matrix} \right]$

17. Matrices $A$ and $B$ will be inverse of each other only if

  1. $AB=BA$

  2. $AB=0,BA=I$

  3. $AB=BA=0$

  4. $AB=BA=I$

Ans: Since, if $A$ is a square matrix of order $m$ , and if there exists another square matrix $B$ of the same order $m$ , such that $AB=BA=I$ , then $B$ is said to be the inverse of $A$. In such a case, it is clear that $A$ is the inverse of $B$.

Thus, matrices $A$ and $B$ will be inverse of each other only if $AB=BA=I$.

Thus, option (D) is correct. (Not in the current syllabus)


Miscellaneous Solutions

1. Let \[A=\left[ \begin{matrix}   0 & 1  \\   0 & 0  \\ \end{matrix} \right]\] , show that \[{{\left( aI+bA \right)}^{n}}={{a}^{n}}I+n{{a}^{n-1}}bA\] , where \[I\] is the identity matrix of order \[2}\] and \[n\in N\].

Ans: Given \[A=\left[ \begin{matrix}   0 & 1  \\   0 & 0  \\ \end{matrix} \right]\]

By using the principle of mathematical induction.

For \[n=1\] ,

\[P\left( 1 \right):{{\left( aI+bA \right)}^{1}}={{a}^{1}}I+{{a}^{0}}bA\]

\[\Rightarrow P\left( 1 \right):\left( aI+bA \right)=aI+bA\]

Therefore, the result is true for \[n=1\] .

Let the result be true for \[n=k\] .

That is, \[P\left( k \right):{{\left( aI+bA \right)}^{k}}={{a}^{k}}I+k{{a}^{k-1}}bA\]

Now, we have to prove that the result is true for \[n=k+1\] .

\[{{\left( aI+bA \right)}^{k-1}}={{\left( aI+bA \right)}^{k}}\left( aI+bA \right)\]

\[\Rightarrow {{\left( aI+bA \right)}^{k-1}}=\left( {{a}^{k}}I+k{{a}^{k-1}}bA \right)\left( aI+bA \right)\]

\[\Rightarrow {{\left( aI+bA \right)}^{k-1}}={{a}^{k}}I+\left( k+1 \right){{a}^{k}}bA+k{{a}^{k-1}}{{b}^{2}}{{A}^{2}}\]                        …(1)

Now, 

\[{{A}^{2}}=\left[ \begin{matrix}   0 & 1  \\   0 & 0  \\ \end{matrix} \right]\left[ \begin{matrix}   0 & 1  \\   0 & 0  \\ \end{matrix} \right]\]

\[{{A}^{2}}=\left[ \begin{matrix}   0 & 0  \\  0 & 0  \\ \end{matrix} \right]\]

\[{{A}^{2}}=0\]

\[\Rightarrow {{\left( aI+bA \right)}^{k-1}}={{a}^{k}}I+\left( k+1 \right){{a}^{k}}bA\]

Thus, the result is true for \[n=k+1\]

Therefore, by the principal of mathematical induction, we have:

\[{{\left( aI+bA \right)}^{n}}={{a}^{n}}I+n{{a}^{n-1}}bA\] where \[A=\left[ \begin{matrix}   0 & 1  \\   0 & 0  \\ \end{matrix} \right]\] , \[n\in N\].


2. If \[A=\left[ \begin{matrix}  1 & 1 & 1  \\   1 & 1 & 1  \\   1 & 1 & 1  \\ \end{matrix} \right]\] , prove that \[{{A}^{n}}=\left[ \begin{matrix}   {{3}^{n-1}} & {{3}^{n-1}} & {{3}^{n-1}}  \\   {{3}^{n-1}} & {{3}^{n-1}} & {{3}^{n-1}}  \\   {{3}^{n-1}} & {{3}^{n-1}} & {{3}^{n-1}}  \\ \end{matrix} \right]\] , \[n\in N\].

Ans: Given \[A=\left[ \begin{matrix}   1 & 1 & 1  \\   1 & 1 & 1  \\   1 & 1 & 1  \\ \end{matrix} \right]\]

By using the principles of mathematical induction.

For \[n=1\] , we have

\[P\left( 1 \right)=\left[ \begin{matrix}   {{3}^{1-1}} & {{3}^{1-1}} & {{3}^{1-1}}  \\   {{3}^{1-1}} & {{3}^{1-1}} & {{3}^{1-1}}  \\   {{3}^{1-1}} & {{3}^{1-1}} & {{3}^{1-1}}  \\ \end{matrix} \right]\]

\[\Rightarrow P\left( 1 \right)=\left[ \begin{matrix}   {{3}^{0}} & {{3}^{0}} & {{3}^{0}}  \\   {{3}^{0}} & {{3}^{0}} & {{3}^{0}}  \\   {{3}^{0}} & {{3}^{0}} & {{3}^{0}}  \\ \end{matrix} \right]\]

\[\Rightarrow P\left( 1 \right)=\left[ \begin{matrix}   1 & 1 & 1  \\   1 & 1 & 1  \\   1 & 1 & 1  \\ \end{matrix} \right]\]

\[\Rightarrow P\left( 1 \right)=A\]

Therefore, the result is true for \[n=1\] .

Let the result be true for \[n=k\] .

\[P\left( k \right):{{A}^{k}}=\left[ \begin{matrix}   {{3}^{k-1}} & {{3}^{k-1}} & {{3}^{k-1}}  \\   {{3}^{k-1}} & {{3}^{k-1}} & {{3}^{k-1}}  \\   {{3}^{k-1}} & {{3}^{k-1}} & {{3}^{k-1}}  \\ \end{matrix} \right]\]

Now, we have to prove that the result is true for \[n=k+1\] .

Now, \[{{A}^{k+1}}=A\cdot {{A}^{k}}\]

\[\Rightarrow {{A}^{k+1}}=\left[ \begin{matrix}   1 & 1 & 1  \\   1 & 1 & 1  \\   1 & 1 & 1  \\ \end{matrix} \right]\left[ \begin{matrix}   {{3}^{k-1}} & {{3}^{k-1}} & {{3}^{k-1}}  \\   {{3}^{k-1}} & {{3}^{k-1}} & {{3}^{k-1}}  \\   {{3}^{k-1}} & {{3}^{k-1}} & {{3}^{k-1}}  \\ \end{matrix} \right]\]

\[\Rightarrow {{A}^{k+1}}=\left[ \begin{matrix}   3\cdot {{3}^{k-1}} & 3\cdot {{3}^{k-1}} & 3\cdot {{3}^{k-1}}  \\   3\cdot {{3}^{k-1}} & 3\cdot {{3}^{k-1}} & 3\cdot {{3}^{k-1}}  \\   3\cdot {{3}^{k-1}} & 3\cdot {{3}^{k-1}} & 3\cdot {{3}^{k-1}}  \\ \end{matrix} \right]\]

\[\Rightarrow {{A}^{k+1}}=\left[ \begin{matrix}   {{3}^{\left( k+1 \right)-1}} & {{3}^{\left( k+1 \right)-1}} & {{3}^{\left( k+1 \right)-1}}  \\   {{3}^{\left( k+1 \right)-1}} & {{3}^{\left( k+1 \right)-1}} & {{3}^{\left( k+1 \right)-1}}  \\   {{3}^{\left( k+1 \right)-1}} & {{3}^{\left( k+1 \right)-1}} & {{3}^{\left( k+1 \right)-1}}  \\ \end{matrix} \right]\]

Thus, the result is true for \[n=k+1\]

Therefore, by the principal of mathematical induction, we have:

 \[{{A}^{n}}=\left[ \begin{matrix}   {{3}^{n-1}} & {{3}^{n-1}} & {{3}^{n-1}}  \\  {{3}^{n-1}} & {{3}^{n-1}} & {{3}^{n-1}}  \\   {{3}^{n-1}} & {{3}^{n-1}} & {{3}^{n-1}}  \\ \end{matrix} \right]\] where \[A=\left[ \begin{matrix}   0 & 1  \\   0 & 0  \\ \end{matrix} \right]\] , \[n\in N\].


3. If \[A=\left[ \begin{matrix}   3 & -4  \\   1 & -1  \\ \end{matrix} \right]\] , then prove \[{{A}^{n}}=\left[ \begin{matrix}   1+2n & -4n  \\   n & 1-2n  \\ \end{matrix} \right]\] where \[n\] is any positive integer.

Ans: Given \[A=\left[ \begin{matrix}   3 & -4  \\   1 & -1  \\ \end{matrix} \right]\]

By using the principle of mathematical induction.

For \[n=1\] ,

\[P\left( 1 \right):A=\left[ \begin{matrix}   3 & -4  \\   1 & -1  \\ \end{matrix} \right]\]

Therefore, the result is true for \[n=1\] .

Let the result be true for \[n=k\] .

That is, \[P\left( k \right):{{A}^{k}}=\left[ \begin{matrix}   1+2k & -4k  \\   k & 1-2k  \\ \end{matrix} \right]\] , \[n\in N\]

Now, we have to prove that the result is true for \[n=k+1\] .

\[{{A}^{k+1}}=A\cdot {{A}^{k}}\]

\[\Rightarrow {{A}^{k+1}}=\left[ \begin{matrix}   1+2k & -4k  \\   k & 1-2k  \\ \end{matrix} \right]\left[ \begin{matrix}   3 & -4  \\   1 & -1  \\ \end{matrix} \right]\]

\[\Rightarrow {{A}^{k+1}}=\left[ \begin{matrix}   3+2k & -4-4k  \\  1+k & -1-2k  \\ \end{matrix} \right]\]

\[\Rightarrow {{A}^{k+1}}=\left[ \begin{matrix}   1+2\left( k+1 \right) & -4\left( k+1 \right)  \\   1+k & 1-2\left( k+1 \right)  \\ \end{matrix} \right]\]

Thus, the result is true for \[n=k+1\]

Therefore, by the principal of mathematical induction, we have:

\[{{A}^{n}}=\left[ \begin{matrix}   1+2n & -4n  \\   n & 1-2n  \\ \end{matrix} \right]\] , \[n\in N\].


4. If \[A\] and \[B\] are symmetric matrices, prove that \[AB-BA\] is a skew symmetric matrix.

Ans: \[A\] and \[B\] are symmetric matrix, therefore, we have:

\[A'=A\] and \[B'=B\]                                                    …(1)

Here, \[\left( AB-BA \right)'=\left( AB \right)'-\left( BA \right)'\]

\[\Rightarrow \left( AB-BA \right)'=B'A'-A'B'\]

From (1),

\[\Rightarrow \left( AB-BA \right)'=BA-AB\]

\[\Rightarrow \left( AB-BA \right)'=-\left( AB-BA \right)\]                           …(2)

From equation (2),

\[AB-BA\] is a skew symmetric matrix.


5. Show that the matrix \[B'AB\] is symmetric or skew symmetric accordingly as \[A\] is symmetric or skew symmetric.

Ans: Let \[A\] be a symmetric matrix, then \[A'=A\]               …(1)

\[\left( B'AB \right)'=\left\{ B'\left( AB \right) \right\}'\]

\[\Rightarrow \left( B'AB \right)'=\left( AB \right)'B'\]

\[\Rightarrow \left( B'AB \right)'=B'\left( A'B \right)\]

From (1),

\[\Rightarrow \left( B'AB \right)'=B'\left( AB \right)\]

Thus, if \[A\] is a symmetric matrix, then \[B'AB\] is a symmetric matrix.

Let \[A\] be a skew symmetric matrix, then \[A'=-A\]               …(2)

\[\left( B'AB \right)'=\left\{ B'\left( AB \right) \right\}'\]

\[\Rightarrow \left( B'AB \right)'=\left( AB \right)'B'\]

\[\Rightarrow \left( B'AB \right)'=B'\left( A'B \right)\]

From (2),

\[\Rightarrow \left( B'AB \right)'=B'\left( -AB \right)\]

\[\Rightarrow \left( B'AB \right)'=-B'AB\]

Thus, if \[A\] is a skew-symmetric matrix, then \[B'AB\] is a skew-symmetric matrix.

Therefore, the matrix \[B'AB\] is symmetric or skew symmetric accordingly as \[A\] is symmetric or skew symmetric.


6. Solve the system of linear equations, using matrix method.

\[2x-y=-2\]

\[3x+4y=3\]

Ans: The given system of equation can be written in the form of \[AX=B\] where,

\[A=\left[ \begin{matrix}   2 & -1  \\   3 & 4  \\ \end{matrix} \right],X=\left[ \begin{matrix}   x  \\   y  \\ \end{matrix} \right],B=\left[ \begin{matrix}   -2  \\  3  \\ \end{matrix} \right]\]

Now, \[\left| A \right|=8+3=11\]

\[\because \left| A \right|\ne 0\] ,therefore its inverse exists.

We know that,

\[{{A}^{-1}}=\dfrac{1}{\left| A \right|}adjA\]

\[\therefore {{A}^{-1}}=\dfrac{1}{11}adj\left[ \begin{matrix}   4 & 1  \\   -3 & 2  \\ \end{matrix} \right]\]

We can write \[X={{A}^{-1}}B\]

\[\therefore X=\dfrac{1}{11}\left[ \begin{matrix}   4 & 1  \\   -3 & 2  \\ \end{matrix} \right]\left[ \begin{matrix}   -2  \\   3  \\ \end{matrix} \right]\]

\[\Rightarrow \left[ \begin{matrix}   x  \\   y  \\ \end{matrix} \right]=\dfrac{1}{11}\left[ \begin{matrix}   -5  \\   12  \\ \end{matrix} \right]\]

\[\Rightarrow \left[ \begin{matrix}   x  \\   y  \\ \end{matrix} \right]=\left[ \begin{matrix}   \dfrac{-5}{11}  \\   \dfrac{12}{11}  \\ \end{matrix} \right]\]

Thus, \[x=\dfrac{-5}{11}\] and \[y=\dfrac{12}{11}\].


7. For what values of \[x\] , \[\left[ \begin{matrix}   1 & 2 & 1  \\ \end{matrix} \right]\left[ \begin{matrix}   1 & 2 & 0  \\   2 & 0 & 1  \\   1 & 0 & 2  \\ \end{matrix} \right]\left[ \begin{matrix}   0  \\   2  \\   x  \\ \end{matrix} \right]=O\] ?

Ans: Given \[\left[ \begin{matrix}   1 & 2 & 1  \\ \end{matrix} \right]\left[ \begin{matrix}   1 & 2 & 0  \\   2 & 0 & 1  \\   1 & 0 & 2  \\ \end{matrix} \right]\left[ \begin{matrix}   0  \\   2  \\   x  \\ \end{matrix} \right]=O\]

\[\Rightarrow \left[ \begin{matrix}   1+4+1 & 2+0+0 & 0+2+2  \\ \end{matrix} \right]\left[ \begin{matrix}   0  \\   2  \\   x  \\ \end{matrix} \right]=O\]

\[\Rightarrow \left[ \begin{matrix}   6 & 2 & 4  \\\end{matrix} \right]\left[ \begin{matrix}   0  \\   2  \\   x  \\ \end{matrix} \right]=O\]

\[\Rightarrow \left[ 6\left( 0 \right)+2\left( 2 \right)+4\left( x \right) \right]=O\]

\[\Rightarrow \left[ 4+4x \right]=\left[ 0 \right]\]

\[\Rightarrow 4+4x=0\]

\[\therefore x=-1\]

Thus, the required value of \[x\] is \[-1\] .


8. If \[A=\left[ \begin{matrix}   3 & 1  \\   -1 & 2  \\ \end{matrix} \right]\] , show that \[{{A}^{2}}-5A+7I=O\] .

Ans: Given \[A=\left[ \begin{matrix}   3 & 1  \\   -1 & 2  \\ \end{matrix} \right]\]

\[\therefore {{A}^{2}}=A\cdot A\]

\[\Rightarrow {{A}^{2}}=\left[ \begin{matrix}   3 & 1  \\   -1 & 2  \\ \end{matrix} \right]\left[ \begin{matrix}   3 & 1  \\   -1 & 2  \\ \end{matrix} \right]\]

\[\Rightarrow {{A}^{2}}=\left[ \begin{matrix}   3\left( 3 \right)+1\left( -1 \right) & 3\left( 1 \right)+1\left( 2 \right)  \\   -1\left( 3 \right)+2\left( -1 \right) & -1\left( 1 \right)+2\left( 2 \right)  \\ \end{matrix} \right]\]

\[\Rightarrow {{A}^{2}}=\left[ \begin{matrix}   9-1 & 3+2  \\   -3-2 & -1+4  \\ \end{matrix} \right]\]

\[\Rightarrow {{A}^{2}}=\left[ \begin{matrix}   8 & 5  \\   -5 & 3  \\ \end{matrix} \right]\]

LHS: \[{{A}^{2}}-5A+7I\]

\[\therefore \left[ \begin{matrix}   8 & 5  \\   -5 & 3  \\ \end{matrix} \right]-5\left[ \begin{matrix}  3 & 1  \\   -1 & 2  \\ \end{matrix} \right]+7\left[ \begin{matrix}   1 & 0  \\   0 & 1  \\ \end{matrix} \right]\]

\[\Rightarrow \left[ \begin{matrix}   8 & 5  \\   -5 & 3  \\ \end{matrix} \right]-\left[ \begin{matrix}   15 & 5  \\   -5 & 10  \\ \end{matrix} \right]+\left[ \begin{matrix}   7 & 0  \\   0 & 7  \\ \end{matrix} \right]\]

\[\Rightarrow \left[ \begin{matrix}   0 & 0  \\   0 & 0  \\ \end{matrix} \right]\] 

\[\Rightarrow O\]

RHS: \[\Rightarrow O\]

\[LHS=RHS\]

\[\therefore {{A}^{2}}-5A+7I=O\] hence proved.


9. Find \[X\] , if \[\left[ \begin{matrix}   x & -5 & -1  \\ \end{matrix} \right]\left[ \begin{matrix}   1 & 0 & 2  \\   0 & 2 & 1  \\   2 & 0 & 3  \\ \end{matrix} \right]\left[ \begin{matrix}   x  \\   4  \\   1  \\ \end{matrix} \right]=O\] .

Ans: Given \[\left[ \begin{matrix}   x & -5 & -1  \\ \end{matrix} \right]\left[ \begin{matrix}   1 & 0 & 2  \\   0 & 2 & 1  \\   2 & 0 & 3  \\ \end{matrix} \right]\left[ \begin{matrix}   x  \\   4  \\   1  \\ \end{matrix} \right]=O\]

\[\Rightarrow \left[ \begin{matrix}   x-2 & -10 & 2x-8  \\ \end{matrix} \right]\left[ \begin{matrix}   x  \\   4  \\   1  \\ \end{matrix} \right]=O\]

\[\Rightarrow \left[ x\left( x-2 \right)-40+2x-8 \right]=O\]

\[\Rightarrow \left[ {{x}^{2}}-48 \right]=\left[ 0 \right]\]

\[\Rightarrow {{x}^{2}}-48=0\]

\[\therefore x=\pm 4\sqrt{3}\]

Thus, the required value of \[x\] is \[\pm 4\sqrt{3}\] .


10. A manufacture produces three products \[X,Y,Z\] which he sells in two markets.

Annual sales are indicated below:

Market

Products

\[I\]

\[10000\]

\[2000\]

\[18000\]

\[II\]

\[6000\]

\[20000\]

\[8000\]

  1. If unit sale prices of \[X,Y,Z\] are Rs \[2.50\] , Rs \[1.50\] and Rs \[1.00\] , respectively, find the total revenue in each market with the help of matrix algebra.

Ans: Here the total revenue in market \[I\] can be represented in the form of a matrix as:

\[\left[ \begin{matrix}   10000 & 2000 & 18000  \\ \end{matrix} \right]\left[ \begin{matrix}   2.50  \\   1.50  \\   1.00  \\ \end{matrix} \right]\]

\[\Rightarrow 1000\times 2.50+2000\times 1.50+18000\times 1.00\]

\[\Rightarrow 46000\]

And, the total revenue in market \[II\] can be represented in the form of a matrix as:

\[\left[ \begin{matrix}   6000 & 20000 & 8000  \\ \end{matrix} \right]\left[ \begin{matrix}   2.50  \\   1.50  \\   1.00  \\ \end{matrix} \right]\]

\[\Rightarrow 6000\times 2.50+20000\times 1.50+8000\times 1.00\]

\[\Rightarrow 53000\]

Thus, the total revenue in market \[I\] is Rs \[46000\] and the same in market \[II\] is Rs \[53000\].

  1. If the unit costs of the above three commodities are Rs \[2.00\] , Rs \[1.00\] and \[50\] paise respectively. Find the gross profit.

Ans: Here, the total cost prices of all the products in the market \[I\] can be represented in the form of a matrix as:

\[\left[ \begin{matrix}   10000 & 2000 & 18000  \\ \end{matrix} \right]\left[ \begin{matrix}   2.00  \\   1.00  \\   0.50  \\ \end{matrix} \right]\]

\[\Rightarrow 1000\times 2.00+2000\times 1.00+18000\times 0.50\]

\[\Rightarrow 31000\]

As the total revenue in market \[I\] is Rs \[46000\] , the gross profit in this market is \[Rs46000-Rs31000=Rs15000\] .

And, the total revenue in market \[II\] can be represented in the form of a matrix as:

\[\left[ \begin{matrix}   6000 & 20000 & 8000  \\ \end{matrix} \right]\left[ \begin{matrix}   2.00  \\   1.00  \\   0.50  \\ \end{matrix} \right]\]

\[\Rightarrow 6000\times 2.00+20000\times 1.00+8000\times 0.50\]

\[\Rightarrow 36000\]

As the total revenue in market \[II\] is Rs \[53000\] , the gross profit in this market is \[Rs53000-Rs36000=Rs17000\] .


11. Find the matrix \[X\] so that \[X\left[ \begin{matrix}   1 & 2 & 3  \\   4 & 5 & 6  \\ \end{matrix} \right]=\left[ \begin{matrix}   -7 & -8 & -9  \\   2 & 4 & 6  \\ \end{matrix} \right]\].

Ans: Given \[X\left[ \begin{matrix}   1 & 2 & 3  \\   4 & 5 & 6  \\ \end{matrix} \right]=\left[ \begin{matrix}   -7 & -8 & -9  \\   2 & 4 & 6  \\ \end{matrix} \right]\]

Here, \[X\] has to be a \[2\times 2\] matrix.

Now, let \[X=\left[ \begin{matrix}   a & c  \\   b & d  \\ \end{matrix} \right]\]

Thus, we have:

\[\left[ \begin{matrix}   a & c  \\   b & d  \\ \end{matrix} \right]\left[ \begin{matrix}   1 & 2 & 3  \\   4 & 5 & 6  \\ \end{matrix} \right]=\left[ \begin{matrix}   -7 & -8 & -9 \\   2 & 4 & 6  \\ \end{matrix} \right]\]

\[\Rightarrow \left[ \begin{matrix}   a+4c & 2a+5c & 3a+6c  \\   b+4d & 2b+5d & 3b+6d  \\ \end{matrix} \right]=\left[ \begin{matrix}   -7 & -8 & -9  \\   2 & 4 & 6  \\ \end{matrix} \right]\]

Comparing the corresponding elements of two matrices, we have:

\[a+4c=-7\]

\[2a+5c=-8\]

\[3a+6c=-9\]

\[b+4d=2\]

\[2b+5d=4\]

\[3b+6d=6\]

Now, solving the above equations we get,

\[\therefore a=1,b=2,c=-2,d=0\]

Thus, the required matrix \[X\] is \[\left[ \begin{matrix}   1 & -2  \\   2 & 0  \\ \end{matrix} \right]\].


12. If \[A\] and \[B\] are square matrices of the same order such that \[AB=BA\] , then prove by induction that \[A{{B}^{n}}={{B}^{n}}A\] . Further, prove that \[{{\left( AB \right)}^{n}}={{B}^{n}}{{A}^{n}}\] for all \[n\in N\].

Ans: Given, \[A\] and \[B\] are square matrices of the same order such that \[AB=BA\] .

For \[n=1\] , we have:

\[P\left( 1 \right):AB=BA\]                          (Given)

\[\Rightarrow A{{B}^{1}}={{B}^{1}}A\]

Therefore, the result is true for \[n=1\] .

Let the result be true for \[n=k\] .

\[P\left( k \right):A{{B}^{k}}={{B}^{k}}A\]                                        …(1)

Now, we have to prove that the result is true for \[n=k+1\] .

\[A{{B}^{k+1}}=A{{B}^{k}}\cdot B\]

\[\Rightarrow A{{B}^{k+1}}=\left( {{B}^{k}}A \right)B\]

\[\Rightarrow A{{B}^{k+1}}={{B}^{k}}\left( AB \right)\]

\[\Rightarrow A{{B}^{k+1}}={{B}^{k}}\left( BA \right)\]

\[\Rightarrow A{{B}^{k+1}}=\left( {{B}^{k}}B \right)A\]

\[\Rightarrow A{{B}^{k+1}}={{B}^{k+1}}A\]

Therefore, the result is true for \[n=k+1\] .

By the principle of mathematical induction, we have \[A{{B}^{n}}={{B}^{n}}A\] , \[n\in N\].


13. Choose the correct answer in the following questions:

If \[A=\left[ \begin{matrix}   \alpha  & \beta   \\   \gamma  & -\alpha   \\ \end{matrix} \right]\] is such that \[{{A}^{2}}=I\] then 

  1. \[1+{{\alpha }^{2}}+\beta \gamma =0\]

  2. \[1-{{\alpha }^{2}}+\beta \gamma =0\]

  3. \[1-{{\alpha }^{2}}-\beta \gamma =0\]

  4. \[1+{{\alpha }^{2}}-\beta \gamma =0\]

Ans: Given \[A=\left[ \begin{matrix}   \alpha  & \beta   \\   \gamma  & -\alpha  \\ \end{matrix} \right]\] 

\[\therefore {{A}^{2}}=A\cdot A\]

\[\Rightarrow {{A}^{2}}=\left[ \begin{matrix}   \alpha  & \beta   \\   \gamma  & -\alpha   \\ \end{matrix} \right]\left[ \begin{matrix}   \alpha  & \beta   \\   \gamma  & -\alpha   \\ \end{matrix} \right]\]

\[\Rightarrow {{A}^{2}}=\left[ \begin{matrix}   {{\alpha }^{2}}+\beta \gamma  & \alpha \beta -\alpha \beta   \\   \alpha \gamma -\alpha \gamma  & \beta \gamma +{{\alpha }^{2}}  \\ \end{matrix} \right]\]

Now, \[{{A}^{2}}=I\]

\[\Rightarrow \left[ \begin{matrix}   {{\alpha }^{2}}+\gamma  & 0  \\   0 & \beta \gamma +{{\alpha }^{2}}  \\ \end{matrix} \right]=\left[ \begin{matrix}   1 & 0  \\   0 & 1  \\ \end{matrix} \right]\]

Equating the corresponding elements, we get:

\[\beta \gamma +{{\alpha }^{2}}=1\]

\[\Rightarrow {{\alpha }^{2}}+\beta \gamma -1=0\]

\[\Rightarrow 1-{{\alpha }^{2}}-\beta \gamma =0\]


14. If the matrix \[A\] is both symmetric and skew symmetric, then

  1. \[A\] is a diagonal matrix

  2. \[A\] is a zero matrix

  3. \[A\] is a square matrix

  4. None of these

Ans: If a matrix \[A\] is both symmetric and skew symmetric, then

\[A'=A\] and \[A'=-A\]

\[\Rightarrow A=-A\]

\[\Rightarrow A+A=O\]

\[\Rightarrow A=O\]

Thus, option (B) is correct.


15. If \[A\] is a square matrix such that \[{{A}^{2}}=A\] , then \[{{\left( I+A \right)}^{3}}-7A\] is equal to

  1. \[A\]

  2. \[I-A\]

  3. \[I\]

  4. \[3A\]

Ans: \[{{\left( I+A \right)}^{3}}-7A={{I}^{3}}+{{A}^{3}}+3A+3{{A}^{2}}-7A\]

\[\Rightarrow {{\left( I+A \right)}^{3}}-7A=I+{{A}^{3}}+3A+3{{A}^{2}}-7A\]

Given that \[{{A}^{2}}=A\] ,

\[\Rightarrow {{\left( I+A \right)}^{3}}-7A=I+{{A}^{2}}\cdot A+3A+3{{A}^{2}}-7A\]

\[\Rightarrow {{\left( I+A \right)}^{3}}-7A=I+A\cdot A-A\]

\[\Rightarrow {{\left( I+A \right)}^{3}}-7A=I\]

Thus, option (C) is correct.


NCERT Solutions for Class 12 Maths – Free PDF Download

Class 12 Maths Chapter 3 NCERT solutions are now available online in PDF format. One can find the proper answers of Matrices Class 12 here, which have been developed by accomplished mentors complimenting their dexterity. The Class 12 Maths Ch 3 NCERT solutions PDF is uploaded on the official website. Download the file and start solving it right away.


Chapter 3 – Matrices 

3.1 Introduction

In NCERT Class 12 Maths Chapter 3 PDF, students will explore concepts of matrices. The learners can clearly understand the necessary application of matrices in various branches of mathematics. This mathematical tool simplifies the work to a great extent as compared to other straight forward methods. The evolution of the concept of Matrices 12th is the result of an attempt to extract compact and simple techniques of solving systems of linear equations. In this chapter, students will be better acquainted with the basics of matrix and matrix algebra. The utility of matrices expands far beyond the representation of the coefficients in the linear equations system.


Electronic spreadsheet programs for personal computers use Matrix notation and operations.


These, in turn, are used in different areas of business and science like budgeting, sales projection, cost estimation, analyzing the results of an experiment, etc. Many physical operations, such as magnification and reflection through a plane can be presented mathematically with the help of matrices. They are also used in cryptography. Good knowledge of matrices is important in varied individual branches of sciences and also in genetics, economics, sociology, modern psychology and industrial management.

3.2 Matrix

In this section of the chapter,  the fundamental principles of matrices are discussed. With the help of examples cited by established academicians, students can understand the whole concept. Get the proper definition of Matrix Class 12 and learn the order of a matrix considering only those matrices whose elements are real numbers or functions taking actual values. Students will get to know that the horizontal and vertical lines of entries in a matrix are called rows and columns respectively. This section also talks about the size of a matrix by the number of rows and columns it contains.

3.3 Types of Matrices

This part of the chapter delves into different types of matrices illustrated with various examples. It covers column matrix, row matrix, square matrix, diagonal matrix, scalar matrix, identity matrix and zero matrices. It also explores the equality of matrices accompanied with examples and solutions. The questions that have been provided will induce the sensibility of a learner to help them delve deeper into the aspects concerning the NCERT solution of Maths Class 12.

3.4 Operations on Matrices

Matrix Class 12 NCERT Solutions introduces certain operations on matrices, namely, the addition of matrices, multiplication of a matrix by a scalar, differences and multiplication of matrices. Highlighting properties of matrix addition, scalar multiplication of a matrix, multiplication of matrices, etc., students can get a profound understanding of how matrices operate.

3.5. Transpose of a Matrix

This section will focus on transposing a matrix and unique types of matrices such as symmetric and skew-symmetric matrices. It will establish the foundations of the transpose of the matrix with compelling examples and properties. This chapter engraves the basics with the help of illustrated examples and solutions.

3.6 Symmetric and Skew Symmetric Matrices

In this section, the chapter will assist in learning symmetric and skew-symmetric matrices. One can also comprehend the difference between a symmetric matrix and a skew-symmetric matrix. It is imperative to know about the transpose of a matrix and how to find it. Moreover, it is essential to remember all the parts of Chapter 3 that have been covered up until now. This section also explores theorems proving the nature of the symmetric matrix and skew-symmetric matrix.

3.7 Elementary Operation (Transformation) of a Matrix (Not in the current syllabus)

Above section of the chapter explains the six operations (transformations) on a matrix, three of which are due to rows and three due to columns. These are known as elementary operations or changes.

3.8 Invertible Matrices (Not in the current syllabus)

Learn the properties of invertible matrices. This chapter highlights the process of finding a matrix's inverse is known as matrix inversion. It is essential to know that not all matrices are invertible. For a matrix to be invertible, it must be able to be multiplied by its inverse. Furthermore, one can learn how to invert a matrix by elementary operations, shaping a better idea with diverse examples. (Not in the current syllabus)


Some Important Points to remember

1. A matrix is said to be an ordered rectangular array of numbers or functions. These numbers or functions in the array are called the elements or the entries of the matrix.


2. Order of a Matrix

The order of the matrix determines the dimension of the matrix and the number of rows and columns in the matrix. The general representation of matrix order is Amxn, where m is the number of rows and n is the number of columns in the matrix.


3. Types of Matrices

  • Column Matrix: A column matrix is a matrix with only one column.

  • Row Matrix: A row matrix is a matrix with only one row.

  • Square Matrix: A square matrix is one that has an equal number of rows and columns.

  • Diagonal Matrix: A diagonal matrix is a square matrix in which all of the entries except the diagonal elements are zeroes.

  • Scalar Matrix: A scalar matrix is a diagonal matrix in which all diagonal members are the same (non-zero). 

  • Identity Matrix: An identity matrix is a diagonal matrix in which all diagonal members are '1' and all non-diagonal elements are zero. It is represented by the letter I. 

  • Zero or Null Matrix: If all of the elements in a matrix are 0, it is said to be a zero or null matrix.


4. Equality of Matrices: Let A and B be two matrices. These matrices will be equal, if

(i) orders of A and B will be the same

(ii) corresponding elements of two matrices are the same


5. Operations on Matrices

  • Addition of Matrices: The addition of matrices is one of the operations on matrices. By adding the corresponding matrices' elements, two or more matrices of the same order can be added. 

  • Subtraction of Matrices: Matrix subtraction is a matrix operation that performs element-wise subtraction of matrices of the same order, that is, matrices with the same number of rows and columns. When we subtract two matrices, we subtract the elements in each row and column from the respective elements in another matrix's row and column.

  • Multiplication of Matrices: Matrix multiplication is a matrix operation done on two matrices when they are both compatible. The primary condition for multiplying two matrices is that the number of columns in the first matrix equals the number of rows in the second matrix, so the order of the matrices becomes important.


6. Properties of Multiplication of Matrices

  1. Non-commutativity: 

Matrix multiplication will be not commutative i.e. if AB & BA are both defined, then it is not mandatory that AB ≠ BA.

  1. Associative law:

For three matrices A, B, and C, if multiplication is defined, then we can write it as A (BC) = (AB) C.

  1. Multiplicative identity: 

For any square matrix A, there will be an identity matrix of the same order in which  IA = AI = A.


NCERT Solutions For Class 12 Maths Other Chapters PDF Download


Key Features of NCERT Solutions for Class 12 Maths Chapter 3

The chapter should be revised thoroughly to attain good marks in the exam. If a student adheres to the Matrices Class 12 NCERT solutions that have been developed by the experts, then they will learn the foundational base and the core concepts of the topic. The key features are provided as follows:

  • Matrices Class 12 Solutions are crafted elaborately to assist students in the most uncomplicated fashion, easy to study and learn the chapter.

  • With the Matrices Class 12 answers, students will be able to implant solid principles required to solve the exercises much quicker.

  • Class 12 Maths NCERT Solutions Chapter 3 will always help any student facing doubts to clear them and understand promptly.

  • Students should follow the instructions provided in the solutions that will help them attain more marks.


CBSE Class 12 Maths Chapter 3 Other Study Materials


Conclusion

Vedantu's NCERT Solutions for Class 12 Maths Chapter 3 on matrices provide comprehensive and expert guidance for students. These solutions cover all the topics in a thorough and organized manner, ensuring a clear understanding of the subject. Vedantu's solutions are designed to help students ace their exams by providing step-by-step explanations, solved examples, and practice questions. With Vedantu's NCERT Solutions, students can gain confidence in their mathematical skills and enhance their problem-solving abilities. Whether it's understanding matrix operations, determinants, Vedantu's NCERT Solutions for Class 12 Maths Chapter 3 offer valuable support for students aiming for academic success.

FAQs on NCERT Solutions for Class 12 Maths Chapter 3 - Matrices

1. What are the main topics and subtopics of the chapter Matrices?

Matrices are one of the easiest chapters in maths, which, when understood, would become fun to solve. The concepts are as follows: Applications, Matrices, Order of matrices, Types of matrices includes Column matrix, Row matrix, Square matrix, Diagonal matrix, Scalar matrix, Identity matrix, Zero matrix and Equal matrix. You will also read about Operations of matrices which includes Addition of matrices, Subtraction of matrices and Multiplication of matrices. 


The chapter also explains about various laws which are: Commutative law, Associative law, Distributive law, Existence of multiplicative identity and Cancellation law. You will also get an idea of Transpose of a matrix, Properties of transpose, Symmetric and skew matrices and Invertible matrices.

2. Give me a glimpse of the chapter Matrices?

Matrices are majorly rectangular arrays of numbers which are represented in rows and columns. Basically, matrices are used to perform various mathematical operations such as addition, multiplication, subtraction and division. Representing data related to population, infant mortality rate, etc. are some of the widely used areas where matrices are implemented to simplify the calculations and ease the representation of data.


Matrices have substantial use in plotting graphs, statistics and various scientific research purposes. The most common and popular application of matrices is in solving linear equations. Matrices are even used to represent the coefficients of a linear equation. Other than that, matrices even find application in 3D maths, where they are used to define the relationship between two coordinate spaces.

3. How do our solutions help you in scoring good marks in the examination?

Start solving easier problems and slowly move to the medium level and then to the complex level. After each and every topic try to solve the questions and check where you are lagging behind. Practice the weaker area of that particular concept as many times as you can. This also helps you in knowing your strength along with the weakness. 


Maths is always a part of our life and it will also be used in our day to day life. The short-cut technique to score well in maths is to practice. Do not mug up the solutions, try to understand them and solve it in your own way and then check where you have gone wrong.

4. Why should we learn about Matrices in NCERT Solutions for Class 12 Maths Chapter 3?

Matrices are a powerful and essential tool in Mathematics. They are rectangular arrays of numbers represented in the form of rows and columns. It is used to perform several mathematical operations like addition, subtraction, multiplication, and division. It is widely used in various areas to simplify complex operations like plotting graphs, representing the population, statistics, and in various research papers. It also simplifies the method of solving complex linear equations. 

5. How many exercises are there in the Matrix?

There are four exercises and one miscellaneous exercise in the chapter Matrix. You can refer to the Maths Solutions for Class 12 by Vedantu, where you will find the detailed solution of every question in your Maths book. The solutions are explained in a simple and detailed manner and will clear your doubts. It also includes tips and important points that will help you score high in your exams. 

6. Is Class 12 maths tough?

No, Class 12 Maths is not tough. You can score high by regular practice and strong concepts. Solve every question in your NCERT Maths book, including the solved questions. Refer to Vedantu’s Maths Solution for Class 12. It focuses on strengthening your concepts so that you can solve a variety of questions. Practice some questions daily to improve your solving skills. You can refer to Vedantu’s Revision Class 12 Notes which includes all the important concepts and formulas compiled in one place. 

7. What are Matrices in Maths Class 12?

Matrices are rectangular arrays where data is represented in the form of rows and columns. It is a very easy and scoring topic. This chapter includes:

  • Types of Matrices

  • Operations on Matrices

  • Transpose of Matrices

  • Symmetric and Skew-symmetric Matrices

  • Elementary Operations (Transformation) on Matrices

  • Invertible Matrices

These topics are explained in easy language so that the students can score high. You can practice a variety of questions in this chapter since it is relatively easier. 

8. Why should you refer to Vedantu’s Solutions for Class 12 Maths?

Vedantu’s Solutions for Class 12 Maths are prepared by experienced subject-matter experts and include answers to every question in your NCERT Maths book. It includes all the important formulas, concepts, and theorems that are very important from the exam point of view. The solutions are updated according to the latest guidelines of the CBSE board. This is also very useful for revision before your exam. It includes step-wise solutions that will help you to understand the concepts well. The solution PDFs and other study materials such as important questions and revision notes can also be downloaded from the Vedantu app as well for free of cost.